Формула площадь неправильной фигуры: «Как вычислить площадь фигуры неправильной формы?» – Яндекс.Кью

Содержание

Площади неправильных фигур. — таблицы Tehtab.ru

Площади неправильных фигур.

Площади неправильных плоских фигур можно приблизительно определить, используя планиметр, формулу трапеций, правило средних ординат, формулу Симпсона.

Подобные методы могут быть использованы, например, инженерами для оценки площадей индикаторных диаграмм паровых двигателей, землемерами для оценки площадей земельных участков, кораблестроителями для оценки горизонтальных или поперечных сечений кораблей.

Планиметр.

Это инструмент для непосредственного измерения малых площадей, очерченных неправильной кривой.

Формула трапеций.

Чтобы определить площадь PQRS на рис. выше , необходимо:

— Разделить PS на любое число равных интервалов шириной d каждый (чем больше количество интервалов, тем выше точность).

— Аккуратно измерить ординаты у1, у2, у3 и так далее.

-Площадь SPQRS = d[(у

1+у7)/2 +у23+ у456]

В общем, согласно формуле трапеций: Площадь = (ширина интервала)[1/2(первая + последняя ордината)+(сумма остальных ординат)].

 

 

 

 

 

Правило средних ординат.

Чтобы определить площадь ABCD на рис. выше, необходимо:

— Разделить основание AD на любое количество равных интервалов шириной d каждый (чем больше количество интервалов, тем выше точность).

— Восстановить перпендикуляр из середины каждого интервала (на рис. выше оказаны штриховыми линиями).

-Точно измерить ординаты у1, у2, у3 и так далее.

— Площадь SABCD = d(у123+ у456)

В общем виде, правило средних ординат гласит: Площадь = (ширина интервала)*(сумма средних ординат).

 

 

 

 

Формула Симпсона .

 

Чтобы определить площадь PQRS на рис.выше, необходимо:

— Разделить основание PS на четное количество равных интервалов шириной d каждый (чем больше количество интервалов, тем выше точность).

— Точно измерить ординаты у1, у2, у3 и так далее.

— Площадь SPQRS = (d/3)*[(у17) +4(у24+ у6)+2(у35)]

В общем виде, формула Симпсона: Площадь = 1/3(ширина интервала)[(первая + последняя ордината)+4(сумма четных ординат)+2(сумма нечетных ординат)].

 

 

 

 

Пример. Определение площади под кривой с помощью формулы трапеций, правила средних ординат, формулы Симпсона.

Машина стартует из состояния покоя, и ее скорость измеряется каждую секунду в течение 6 секунд:

Время t [c] 0 1 2 3 4 5 6
Скорость v [м/c] 0 3,0 6,0 9,15 13,5 18,5 25,0

Хотя мы понимаем что увеличение числа разбиений увеличивает точность, но для простоты решения возьмем количество разбиений, равное 7.


Определить расстояние, пройденное за 6 секунд (т.е. площадь под графиком):

а) по формуле трапеций, б) по правилу средних ординат, в)по формуле Симпсона.

а) Формула трапеций.

Время на графике делится на 6 интервалов шириной 1 с и измеряются ординаты. Итак, площадь S определяется формулой

S= 1[(0+25)/2 +3+6+ 9,15+13,5+18,5]=62,65 м

б) Правило средних ординат.

Время на графике делится на 6 интервалов шириной 1 с. На рис. пунктирной линией показаны средние ординаты. Измерена каждая средняя ордината. Итак, площадь S определяется формулой

S= 1(1,5 +4,5+7,75+11,75+16+21,5)=63 м

в) Формула Симпсона .

Время на графике делится на 6 интервалов шириной 1 с и измеряются ординаты. Итак, площадь S определяется формулой:

S= (1/3)*[(0+25) +4(3+9,15+ 18,5)+2(6+13,5)]= 61,2 м.

Т.е. машина проехала в среднем 62,283 м.

Как найти площадь неправильной фигуры

В школьном курсе геометрии ученики в основном считают площади правильных многоугольников. Между тем, для решения множества практических задач нередко приходится иметь дело с неправильными геометрическими фигурами. С этой проблемой человек сталкивается и при определении размеров дачного участка или придомовой территории, и при расчете количества ткани для шитья, и еще во многих случаях. Высчитать площадь неправильной фигуры можно несколькими способами.Вам понадобится

Рассмотрите геометрическую фигуру и определите, какие ее параметры вам известны. Это могут быть длины сторон или углы. В зависимости от заданных параметров и выберите способ определения площади. Например, разделите ее на несколько фигур, формулы вычисления площади которых вы знаете. Один из самых распространенных методов — провести диагонали из одного угла ко всем остальным вершинам. В этом случае вам нужно знать формулу вычисления площади произвольного треугольника. Но никто не запрещает разделить заданную фигуру и на другие многоугольники. Например, при расчете площади пола в комнате с нишей удобнее разделить неправильную фигуру на два прямоугольника или квадрата.

Для определения площади не слишком большой детали можно воспользуйтесь палеткой. Ее можно сделать самому. Отрежьте прямоугольный кусок любого прозрачного пластика. Разделите его на квадраты, площадь которых вам известна — например, 1х1 или 0,5х0,5 см. Линейка и угольник должны быть точными. Наложите палетку на деталь. Сосчитайте полные квадратики, затем — неполные. Количество неполных квадратов разделите на 2 и приплюсуйте результат к числу целых. Чем мельче деления на палетке — тем точнее будет результат.Аналогично можно посчитать и площадь участка. Роль палетки будет выполнять сетка из квадратов со стороной 1х1 м, начерченная на земле или отмеченная колышками с протянутыми между ними шнурами. Можно ограничиться и разметкой территории на полосы. .

С крупными площадями можно поступить и иначе. Возьмите максимально точный план участка или придомовой территории. Определите масштаб. Воспользуйтесь одним из предложенных способов. Затем полученное количество квадратных сантиметров переведите в нужный масштаб.

§ Площадь фигур. Площадь квадрата. Площадь прямоугольника. Площадь сложной фигуры

Две фигуры называют равными, если одну их них можно так наложить на другую, что эти фигуры совпадут.

Площади равных фигур равны. Их периметры тоже равны.

Площадь квадрата

Запомните!

Для вычисления площади квадрата нужно умножить его длину на саму себя.

S = a · a

Пример:

SEKFM = EK · EK

SEKFM = 3 · 3 = 9 см2

Формулу площади квадрата, зная определение степени, можно записать следующим образом:

S = a2

Площадь прямоугольника

Запомните!

Для вычисления площади прямоугольника нужно умножить его длину на ширину.

S = a · b

Пример:

SABCD = AB · BC

SABCD = 3 · 7 = 21 см2


Запомните!

Нельзя вычислять периметр или площадь, если длина и ширина выражены в разных единицах длины.

Обязательно проверяйте, чтобы и длина, и ширина были выражены в одинаковых единицах, то есть обе в см, м и т.д.

Площадь сложных фигур

Запомните!

Площадь всей фигуры равна сумме площадей её частей.

Задача: найти площадь огородного участка.

Так как фигура на рисунке не является ни квадратом, ни прямоугольником, рассчитать её площадь можно используя правило выше.

Разделим фигуру на два прямоугольника, чьи площади мы можем легко рассчитать по известной формуле.

SABCE = AB · BC
SEFKL = 10 · 3 = 30 м2
SCDEF = FC · CD
SCDEF = 7 · 5 = 35 м2

Чтобы найти площадь всей фигуры, сложим площади найденных прямоугольников.
S = SABCE + SEFKL
S = 30 + 35 = 65 м2

Ответ: S = 65 м2 — площадь огородного участка.


Свойство ниже может вам пригодиться при решении задач на площадь.

Запомните!

Диагональ прямоугольника делит прямоугольник на два равных треугольника.

Площадь любого из этих треугольников равна половине площади прямоугольника.

Рассмотрим прямоугольник:

АС — диагональ прямоугольника ABCD. Найдём площадь треугольников ABC и ACD

Вначале найдём площадь прямоугольника по формуле.

SABCD = AB · BC
SABCD = 5 · 4 = 20 см2

S ABC = SABCD : 2

S ABC = 20 : 2 = 10 см2

S ABC = S ACD = 10 см2



Как найти площадь фигуры? Ответ на webmath.ru

Содержание:

Определения

Площадь является одним из основных математических понятий. Она характеризует как плоские, так и поверхностные геометрические объекты.

Определение

Площадью плоской замкнутой фигуры называется величина части плоскости, которая находится внутри указанной фигуры.

Единицей измерения площади плоской фигуры является квадрат со стороной, равной единице. Число, соответствующее площади некоторой фигуры, состоящей из частей, равно сумме чисел, соответствующих площадям этих частей. Измерение площадей треугольников и многоугольников основано на возможности построения равновеликих им прямоугольников.

Площадь произвольной ограниченной плоской фигуры определяется как общий предел площадей описанных и вписанных в нее многоугольников, наибольшие стороны которых по длине стремятся к нулю.

Если фигура имеет площадь, то она называется квадрируемой.

Формулы площади основных геометрических фигур

Площадь треугольника

Чтобы найти площадь треугольника, надо найти полупроизведение двух его сторон на синус угла между ними.{2}$$

Читать дальше: формула площади круга и примеры решений →

Площадь квадрата

Чтобы найти площадь квадрата, надо длину его стороны возвести в квадрат, то есть

Читать дальше: формула площади квадрата и примеры решений →

Площадь прямоугольника

Чтобы найти площадь прямоугольника, надо его длину умножить на ширину, то есть

Читать дальше: формула площади прямоугольника и примеры решений →

Площадь параллелограмма

Чтобы найти площадь параллелограмма, нужно найти произведение стороны $a$ параллелограмма на высоту , проведенную к этой стороне, то есть

Читать дальше: формулы площади параллелограмма и примеры решений →

Площадь трапеции

Чтобы найти площадь трапеции, нужно длину средней линии умножить на длину высоты , опущенной к основанию:

Читать дальше: формулы площади трапеции и примеры решений →

Площадь ромба

Чтобы найти площадь ромба, надо длину стороны умножить на длину высоты, проведенной к этой стороне:

Читать дальше: формулы площади ромба и примеры решений →

Площадь эллипса

Чтобы найти площадь эллипса, нужно найти произведение длин большой и малой полуосей этого эллипса на число $\pi$, то есть

Читать дальше: формула площади эллипса и примеры решений →

Площадь сложных геометрических фигур. Как вычислить площадь фигуры. Формулы площади трапеции

Если вы планируете сделать ремонт самостоятельно, то у вас возникнет необходимость составить смету по строительным и отделочным материалам. Для этого вам понадобится рассчитать площадь помещения, в котором вы планируете произвести ремонтные работы. Главным помощником в этом выступает специально разработанная формула. Площадь помещения, а именно ее расчет, позволит вам сэкономить немалые деньги на строительных материалах и направить высвободившиеся денежные ресурсы в более нужное русло.

Геометрическая форма комнаты

Формула расчета площади помещения напрямую зависит от ее формы. Наиболее типичными для отечественных сооружений являются прямоугольные и квадратные комнаты. Однако в ходе перепланировки стандартная форма может искажаться. Комнаты бывают:

  • Прямоугольные.
  • Квадратные.
  • Сложной конфигурации (например, круглые).
  • С нишами и выступами.

Каждая из них имеет свои особенности расчета, но, как правило, используется одна и та же формула. Площадь помещения любой формы и размера, так или иначе, поддается вычислению.

Помещение прямоугольной или квадратной формы

Чтобы рассчитать площадь комнаты прямоугольной или квадратной формы, достаточно вспомнить школьные уроки геометрии. Поэтому для вас не должно составить особого труда определить площадь помещения. Формула расчета имеет вид:

S комнаты=A*B, где

А — длина помещения.

В — ширина помещения.

Для измерения этих величин вам понадобится обычная рулетка. Чтобы получить наиболее точные расчёты, стоит измерить стену с обеих сторон. Если значения не сходятся, возьмите за основу среднее значение получившихся данных. Но помните, что любые расчёты имеют свои погрешности, поэтому материал стоит закупать с запасом.

Помещение со сложной конфигурацией

Если ваша комната не попадает под определение «типичной», т.е. имеет форму круга, треугольника, многоугольника, то, возможно, для расчетов вам понадобится другая формула. Площадь помещения с такой характеристикой можно попробовать условно разделить на прямоугольные элементы и произвести расчеты стандартным путем. Если такой возможности у вас нет, тогда воспользуйтесь следующими методиками:

  • Формула нахождения площади круга:

S комн.=π*R 2 , где

R — радиус помещения.

  • Формула нахождения площади треугольника:

S комн.= √ (P(P — A) х (Р — В) х (Р — С)), где

Р — полупериметр треугольника.

А, В, С — длины его сторон.

Отсюда Р=А+В+С/2

Если в процессе расчета у вас возникли затруднения, то лучше не мучать себя и обратиться к профессионалам.

Площадь помещения с выступами и нишами

Зачастую стены украшают декоративными элементами в форме всевозможных ниш или выступов. Также их наличие может быть обусловлено необходимостью скрыть некоторые неэстетичные элементы вашей комнаты. Наличие выступов или ниш на вашей стене означает, что расчет следует проводить поэтапно. Т.е. сначала находится площадь ровного участка стены, а затем к нему прибавляется площадь ниши или выступа.

Площадь стены находится по формуле:

S стен = Р х С, где

Р — периметр

С — высота

Также нужно учитывать наличие окон и дверей. Их площадь необходимо отнять от получившегося значения.

Комната с многоуровневым потолком

Многоуровневый потолок не так сильно усложняет расчеты, как это кажется на первый взгляд. Если он имеет простую конструкцию, то можно произвести расчеты по принципу нахождения площади стен, осложненных нишами и выступами.

Однако если конструкция вашего потолка имеет дуго- и волнообразные элементы, то целесообразнее определить его площадь с помощью площади пола. Для этого необходимо:

  1. Найти размеры всех прямых участков стен.
  2. Найти площадь пола.
  3. Перемножить длину и высоту вертикальных участков.
  4. Суммировать получившееся значение с площадью пола.

Пошаговая инструкция по определению общей

площади помещения

  1. Освободите помещение от ненужных вещей. В процессе замеров вам понадобится свободный доступ ко всем участкам вашей комнаты, поэтому нужно избавиться от всего, что может этому препятствовать.
  2. Визуально разделите комнату на участки правильной и неправильной формы. Если ваше помещение имеет строго квадратную или прямоугольную форму, то этот этап можно пропустить.
  3. Сделайте произвольную схему помещения. Этот чертеж нужен для того, чтобы все данные были у вас всегда под рукой. Также он не даст вам возможности запутаться в многочисленных замерах.
  4. Замеры необходимо производить несколько раз. Это важное правило для исключения ошибок в подсчетах. Также если вы используете убедитесь, что луч лежит ровно на поверхности стены.
  5. Найдите общую площадь помещения. Формула общей площади помещения заключается в нахождении суммы всех площадей отдельных участков комнаты. Т.е. S общ.= S стен+S пола+S потолка

Соблюдение Вашей конфиденциальности важно для нас. По этой причине, мы разработали Политику Конфиденциальности, которая описывает, как мы используем и храним Вашу информацию. Пожалуйста, ознакомьтесь с нашими правилами соблюдения конфиденциальности и сообщите нам, если у вас возникнут какие-либо вопросы.

Сбор и использование персональной информации

Под персональной информацией понимаются данные, которые могут быть использованы для идентификации определенного лица либо связи с ним.

От вас может быть запрошено предоставление вашей персональной информации в любой момент, когда вы связываетесь с нами.

Ниже приведены некоторые примеры типов персональной информации, которую мы можем собирать, и как мы можем использовать такую информацию.

Какую персональную информацию мы собираем:

  • Когда вы оставляете заявку на сайте, мы можем собирать различную информацию, включая ваши имя, номер телефона, адрес электронной почты и т.д.

Как мы используем вашу персональную информацию:

  • Собираемая нами персональная информация позволяет нам связываться с вами и сообщать об уникальных предложениях, акциях и других мероприятиях и ближайших событиях.
  • Время от времени, мы можем использовать вашу персональную информацию для отправки важных уведомлений и сообщений.
  • Мы также можем использовать персональную информацию для внутренних целей, таких как проведения аудита, анализа данных и различных исследований в целях улучшения услуг предоставляемых нами и предоставления Вам рекомендаций относительно наших услуг.
  • Если вы принимаете участие в розыгрыше призов, конкурсе или сходном стимулирующем мероприятии, мы можем использовать предоставляемую вами информацию для управления такими программами.

Раскрытие информации третьим лицам

Мы не раскрываем полученную от Вас информацию третьим лицам.

Исключения:

  • В случае если необходимо — в соответствии с законом, судебным порядком, в судебном разбирательстве, и/или на основании публичных запросов или запросов от государственных органов на территории РФ — раскрыть вашу персональную информацию. Мы также можем раскрывать информацию о вас если мы определим, что такое раскрытие необходимо или уместно в целях безопасности, поддержания правопорядка, или иных общественно важных случаях.
  • В случае реорганизации, слияния или продажи мы можем передать собираемую нами персональную информацию соответствующему третьему лицу – правопреемнику.

Защита персональной информации

Мы предпринимаем меры предосторожности — включая административные, технические и физические — для защиты вашей персональной информации от утраты, кражи, и недобросовестного использования, а также от несанкционированного доступа, раскрытия, изменения и уничтожения.

Соблюдение вашей конфиденциальности на уровне компании

Для того чтобы убедиться, что ваша персональная информация находится в безопасности, мы доводим нормы соблюдения конфиденциальности и безопасности до наших сотрудников, и строго следим за исполнением мер соблюдения конфиденциальности.

Площадь геометрической фигуры — численная характеристика геометрической фигуры показывающая размер этой фигуры (части поверхности, ограниченной замкнутым контуром данной фигуры). Величина площади выражается числом заключающихся в нее квадратных единиц.

Формулы площади треугольника

  1. Формула площади треугольника по стороне и высоте
    Площадь треугольника равна половине произведения длины стороны треугольника на длину проведенной к этой стороне высоты
  2. Формула площади треугольника по трем сторонам и радиусу описанной окружности
  3. Формула площади треугольника по трем сторонам и радиусу вписанной окружности
    Площадь треугольника равна произведения полупериметра треугольника на радиус вписанной окружности.
  4. где S — площадь треугольника,
    — длины сторон треугольника,
    — высота треугольника,
    — угол между сторонами и,
    — радиус вписанной окружности,
    R — радиус описанной окружности,

Формулы площади квадрата

  1. Формула площади квадрата по длине стороны
    Площадь квадрата равна квадрату длины его стороны.
  2. Формула площади квадрата по длине диагонали
    Площадь квадрата равна половине квадрата длины его диагонали.
  3. где S — Площадь квадрата,
    — длина стороны квадрата,
    — длина диагонали квадрата.

Формула площади прямоугольника

    Площадь прямоугольника равна произведению длин двух его смежных сторон

    где S — Площадь прямоугольника,
    — длины сторон прямоугольника.

Формулы площади параллелограмма

  1. Формула площади параллелограмма по длине стороны и высоте
    Площадь параллелограмма
  2. Формула площади параллелограмма по двум сторонам и углу между ними
    Площадь параллелограмма равна произведению длин его сторон умноженному на синус угла между ними.

    a · b · sin α

  3. где S — Площадь параллелограмма,
    — длины сторон параллелограмма,
    — длина высоты параллелограмма,
    — угол между сторонами параллелограмма.

Формулы площади ромба

  1. Формула площади ромба по длине стороны и высоте
    Площадь ромба равна произведению длины его стороны и длины опущенной на эту сторону высоты.
  2. Формула площади ромба по длине стороны и углу
    Площадь ромба равна произведению квадрата длины его стороны и синуса угла между сторонами ромба.
  3. Формула площади ромба по длинам его диагоналей
    Площадь ромба равна половине произведению длин его диагоналей.
  4. где S — Площадь ромба,
    — длина стороны ромба,
    — длина высоты ромба,
    — угол между сторонами ромба,
    1 , 2 — длины диагоналей.

Формулы площади трапеции

  1. Формула Герона для трапеции

    Где S — Площадь трапеции,
    — длины основ трапеции,
    — длины боковых сторон трапеции,

Как найти площадь фигуры?


Знать и уметь рассчитывать площади различных фигур необходимо не только для решения простых геометрических задач. Не обойтись без этих знаний и при составлении или проверке смет на ремонт помещений, расчета количества необходимых расходных материалов. Поэтому давайте разберемся, как находить площади разных фигур.

Часть плоскости, заключенная внутри замкнутого контура, называется площадью этой плоскости. Выражается площадь количеством заключенных в ней квадратных единиц.

Чтобы вычислить площадь основных геометрических фигур, необходимо использовать правильную формулу.

Площадь треугольника

Обозначения:
  1. Если известны h, a, то площадь искомого треугольника определяется как произведение длин стороны и высоты треугольника, опущенной к этой стороне, разделенное пополам: S=(a·h)/2
  2. Если известны a, b, c, то искомая площадь рассчитывается по формуле Герона: корень квадратный, взятый из произведения половины периметра треугольника и трех разностей половины периметра и каждой стороны треугольника: S = √(p·(p — a)·(p — b)·(p — c)).
  3. Если известны a, b, γ, то площадь треугольника определяется как половина произведения 2-х сторон, умноженная на значение синуса угла между этими сторонами: S=(a·b·sin γ)/2
  4. Если известны a, b, c, R, то искомая площадь определяется как деление произведения длин всех сторон треугольника на четыре радиуса описанной окружности: S=(a·b·c)/4R
  5. Если известны p, r, то искомая площадь треугольника определяется умножением половины периметра на радиус вписанной в него окружности: S=p·r

Площадь квадрата

Обозначения:
  1. Если известна сторона, то площадь данной фигуры определяется как квадрат длины его стороны: S=a 2
  2. Если известна d, то площадь квадрата определяется как половина квадрата длины его диагонали: S=d 2 /2

Площадь прямоугольника

Обозначения:
  • S — определяемая площадь,
  • a, b — длины сторон прямоугольника.
  1. Если известны a, b, то площадь данного прямоугольника определяется произведением длин двух его сторон: S=a·b
  2. Если длины сторон неизвестны, то площадь прямоугольника нужно разбить на треугольники. В этом случае площадь прямоугольника определяется как сумма площадей составляющих его треугольников.

Площадь параллелограмма

Обозначения:
  • S — искомая площадь,
  • a, b — длины сторон,
  • h — длина высоты данного параллелограмма,
  • d1, d2 — длины двух диагоналей,
  • α — угол, находящийся между сторонами,
  • γ — угол, находящийся между диагоналями.
  1. Если известны a, h, то искомая площадь определяется перемножением длин стороны и высоты, опущенной на эту сторону: S=a·h
  2. Если известны a, b, α, то площадь параллелограмма определяется перемножением длин сторон параллелограмма и значения синуса угла между этими сторонами: S=a·b·sin α
  3. Если известны d 1 , d 2 , γ то площадь параллелограмма определяется как половина произведения длин диагоналей и значения синуса угла между этими диагоналями: S=(d 1 ·d 2 ·sinγ)/2

Площадь ромба

Обозначения:
  • S — искомая площадь,
  • a — длина стороны,
  • h — длина высоты,
  • α — меньший угол между двумя сторонами,
  • d1, d2 — длины двух диагоналей.
  1. Если известны a, h, то площадь ромба определяется умножением длины стороны на длину высоты, которая опущена на эту сторону: S=a·h
  2. Если известны a, α, то площадь ромба определяется перемножением квадрата длины стороны на синус угла между сторонами: S=a 2 ·sin α
  3. Если известны d 1 и d 2 , то искомая площадь определяется как половина произведения длин диагоналей ромба: S=(d 1 ·d 2)/2

Площадь трапеции

Обозначения:
  1. Если известны a, b, c, d, то искомая площадь определяется по формуле: S= (a+b) /2 *√ .
  2. При известных a, b, h, искомая площадь определяется как произведение половины суммы оснований и высоты трапеции: S=(a+b)/2·h

Площадь выпуклого четырехугольника

Обозначения:
  1. Если известны d 1 , d 2 , α, то площадь выпуклого четырехугольника определяется как половина произведения диагоналей четырехугольника, умноженная на величину синуса угла между этими диагоналями: S=(d 1 · d 2 ·sin α)/2
  2. При известных p, r площадь выпуклого четырехугольника определяется как произведение полупериметра четырехугольника на радиус окружности, вписанной в этот четырехугольник: S=p·r
  3. Если известны a, b, c, d, θ, то площадь выпуклого четырехугольника определяется как корень квадратный из произведений разницы полупериметра и длины каждой стороны за минусом произведения длин всех сторон и квадрата косинуса половины суммы двух противоположных углов: S 2 = (p — a)(p — b)(p — c)(p — d) — abcd·cos 2 ((α+β)/2)

Площадь круга

Обозначения:

Если известен r, то искомая площадь определяется как произведение числа π на радиус в квадрате: S=π r 2

Если известна d, то площадь круга определяется как произведение числа π на квадрат диаметра, поделенное на четыре: S=(π·d 2)/4

Площадь сложной фигуры

Сложную можно разбить на простые геометрические фигуры. Площадь сложной фигуры определяется как сумма или разность составляющих площадей. Рассмотрим, к примеру, кольцо.

Обозначение:
  • S — площадь кольца,
  • R, r — радиусы внешней окружности и внутренней соответственно,
  • D, d — диаметры внешней окружности и внутренней соответственно.

Для того чтобы найти площадь кольца, надо из площади большего круга отнять площадь меньшего круга. S = S1-S2 = πR 2 -πr 2 = π (R 2 -r 2).

Таким образом, если известны R и r, то площадь кольца определяется как разница квадратов радиусов внешней и внутренней окружностей, умноженная на число пи: S=π(R 2 -r 2).

Если известны D и d, то площадь кольца определяется как четверть разницы квадратов диаметров внешней и внутренней окружностей, умноженная на число пи: S= (1/4)(D 2 -d 2) π.

Площадь закрашенной фигуры

Предположим, что внутри одного квадрата (А) находится другой (Б) (меньшего размера), и нам нужно найти закрашенную полость между фигурами «А» и «Б». Скажем так, «рамку» маленького квадрата. Для этого:

  1. Находим площадь фигуры «А» (вычисляется по формуле нахождения площади квадрата).
  2. Аналогичным образом находим площадь фигуры «Б».
  3. Вычитаем из площади «А» площадь «Б». И таким образом получаем площадь закрашенной фигуры.

Теперь вы знаете, как находить площади разных фигур.

Площади геометрических фигур — численные значения, характеризующие их размер в двумерном пространстве. Эта величина может измеряться в системных и внесистемных единицах. Так, например, внесистемная единица площади — сотка, гектар. Это в том случае, если измеряемой поверхностью является участок земли. Системная же единица площади — квадрат длины. В системе СИ принято считать, что единица площади плоской поверхности — это квадратный метр. В СГС единица площади выражается через квадратный сантиметр.

Геометрия и формулы площадей неразрывно связаны. Эта связь заключается в том, что вычисление площадей плоских фигур основывается именно на их применении. Для многих фигур выведены несколько вариантов, по которым вычисляются их квадратные размеры. Опираясь на данные из условия задачи, мы можем определить максимально простой способ для решения. Тем самым облегчить расчет и свести вероятность ошибки вычисления к минимуму. Для этого рассмотрим основные площади фигур в геометрии.

Формулы для нахождения площади любого треугольника представлены несколькими вариантами:

1) Площадь треугольника рассчитывается по основанию a и высоте h. Основанием считают сторону фигуры, на которую опущена высота. Тогда площадь треугольника:

2) Площадь прямоугольного треугольника рассчитывается точно также, если гипотенузу считать основанием. Если же за основание принять катет, то площадь прямоугольного треугольника будет равна уменьшенному вдвое произведению катетов.

На этом формулы для вычисления площади любого треугольника не заканчиваются. Другое выражение содержит стороны a,b и синусоидальную функцию угла γ, заключенного между a и b. Значение синуса находится по таблицам. Также его можно узнать с помощью калькулятора. Тогда площадь треугольника:

По данному равенству тоже можно убедиться в том, что площадь прямоугольного треугольника определяется через длины катетов. Т.к. угол γ — прямой, поэтому площадь прямоугольного треугольника рассчитывается без умножения на функцию синуса.

3) Рассмотрим частный случай — правильный треугольник, у которого сторона a известна по условию или ее длина найдется при решении. О фигуре в задаче по геометрии больше ничего не известно. Тогда площадь как найти при этом условии? В этом случае применяется формула для площади правильного треугольника:

Прямоугольник

Как найти площадь прямоугольника и использовать при этом размеры сторон, имеющих общую вершину? Выражение для вычисления такое:

Если для вычисления площади прямоугольника требуется использовать длины диагоналей, то тогда понадобится функция синуса угла, образованного при их пересечении. Такая формула площади прямоугольника имеет вид:

Квадрат

Площадь квадрата определяют как вторую степень длины стороны:

Доказательство вытекает из определения, согласно которому квадратом называют прямоугольник. У всех сторон, образующих квадрат, одинаковые размеры. Поэтому вычисление площади такого прямоугольника сводится к перемножению одной на другую, т. е. ко второй степени стороны. И формула для вычисления площади квадрата примет искомый вид.

Площадь квадрата можно найти другим способом, например, если использовать диагональ:

Как вычислить площадь фигуры, которая образована частью плоскости, ограниченной окружностью? Для расчета площади формулы такие:

Параллелограмм

Для параллелограмма формула содержит линейные размеры стороны, высоты и математическое действие — умножение. Если же высота неизвестна, то тогда как найти площадь параллелограмма? Есть еще один способ вычисления. Потребуется определенное значение, которое примет тригонометрическая функция угла, образованного смежными сторонами, а также их длины.

Формулы площади параллелограмма таковы:

Ромб

Как найти площадь четырехугольника, называемого ромбом? Площадь ромба определяется с помощью простых математических действий с диагоналями. Доказательство опирается на тот факт, что отрезки диагоналей в d1 и d2 пересекаются под прямым углом. По таблице синусов видно, что для прямого угла данная функция равна единице. Поэтому площадь ромба рассчитывается так:

Еще площадь ромба может быть найдена другим способом. Доказать это тоже нетрудно, если учесть, что стороны его одинаковы по длине. Затем подставить их произведение в похожее выражение для параллелограмма. Ведь частным случаем именно этой фигуры является ромб. Здесь γ — внутренний угол ромба. Площадь ромба определяют так:

Трапеция

Как найти площадь трапеции через основания (a и b), если в задаче указаны их длины? Здесь без известного значения длины высоты h вычислить площадь такой трапеции не удастся. Т.к. эту величину содержит выражение для вычисления:

Квадратный размер прямоугольной трапеции тоже можно вычислить таким же способом. При этом учитывают, что в прямоугольной трапеции понятия высоты и боковой стороны объединены. Поэтому для прямоугольной трапеции нужно указывать вместо высоты длину боковой стороны.

Цилиндр и параллелепипед

Рассмотрим что нужно, чтобы рассчитать поверхность всего цилиндра. Площадь данной фигуры составляет пара кругов, называемых основаниями, и боковая поверхность. Окружности, образующие круги имеют длины радиусов, равные r. Для площади цилиндра имеет место такое вычисление:

Как найти площадь параллелепипеда, который состоит из трех пар граней? Его измерения совпадают с конкретной парой. Грани, находящиеся противоположно, имеют одинаковые параметры. Сначала находят S(1), S(2), S(3) — квадратные размеры неравных граней. Затем уже площадь поверхности параллелепипеда:

Кольцо

Две окружности с общим центром образуют кольцо. Они же ограничивают площадь кольца. При этом обе расчетные формулы учитывают размеры каждой окружности. Первая из них, вычисляющая площадь кольца, содержит больший R и меньший r радиусы. Чаще их называют внешним и внутренним. Во втором выражении площадь кольца рассчитывается через больший D и меньший d диаметры. Таким образом, площадь кольца по известным радиусам рассчитывают так:

Площадь кольца, с использованием длин диаметров, определяют следующим образом:

Многоугольник

Как найти площадь многоугольника, форма которого не является правильной? Общей формулы для площади таких фигур нет. Но если она изображена на координатной плоскости, например, это может быть клетчатая бумага, тогда как найти площадь поверхности в этом случае? Тут применяют способ, который не требует приблизительно измерить фигуру. Поступают так: если нашли точки, которые попадают в уголок клетки или имеют целые координаты, то учитывают только их. Чтобы затем выяснить, чему равна площадь, используют формулу, доказанную Пиком. Необходимо сложить количество точек, расположенных внутри ломаной линии с половиной точек, лежащих на ней, и вычесть единицу, т. е. вычисляется это таким образом:

где В,Г — количество точек, расположенных внутри и на всей ломаной линии соответственно.

Площадь неправильного четырехугольника с заданными сторонами — Строительство и стройтехника

Площадь четырехугольника — формулы, примеры расчета

Четырехугольником называется фигура, состоящая из четырех вершин, три из которых не лежат на одной прямой, и отрезков, соединяющих их.

Существует множество четырехугольников. К ним относятся параллелограммы, квадраты, ромбы, трапеции. Найти площадь квадрата можно найти по сторонам, площадь ромба легко вычисляется по диагоналям. В произвольном четырехугольнике также можно использовать все элементы для вывода формулы площади четырехугольника. Для начала рассмотрим формулу площади четырехугольника через диагональ. Для того, чтобы ее использовать потребуются длины диагоналей и размер острого угла между ними. Зная необходимые данные можно проводить пример расчета площади четырехугольника по такой формуле:

Половина произведения диагоналей и синуса острого угла между ними является площадью четырехугольника. Рассмотрим пример расчета площади четырехугольника через диагональ.

Пусть дан четырехугольник с двумя диагоналями d1=5 см;d2=4см. Острый угол между ними равен α = 30°. Формула площади четырехугольника через диагонали легко применяется для известных условий. Подставим данные:

На примере расчета площади четырехугольника через диагонали понимаем, что формула очень похожа на расчет площади параллелограмма.

Площадь четырехугольника по сторонам

Когда известны длины сторон фигуры, можно применить формулу площади четырехугольника по сторонам. Для применения этих расчетов потребуется найти полупериметр фигуры. Мы помним, что периметр – это сумма длин всех сторон. Полупериметр – это половина периметра. В нашем прямоугольнике со сторонами a, b, c, d формула полупериметра будет выглядеть так:
Зная стороны, выводим формулу. Площадь четырехугольника представляет собой корень из произведения разности полупериметра с длиной каждой стороны:

Рассмотрим пример расчета площади четырехугольника через стороны. Дан произвольный четырехугольник со сторонами a = 5 см, b = 4 см, с = 3 см, d = 6 см. Для начала найдем полупериметр:

используем найденное значение для расчета площади:

Площадь четырехугольника, заданного координатами

Формула площади четырехугольника по координатам используется для расчета площади фигур, которые располагаются в системе координат. В этом случае для начала требуется расчет длин необходимых сторон. В зависимости от типа четырехугольника может меняться и сама формула. Рассмотрим пример расчета площади четырехугольника, используя квадрат, который лежит в системе координат XY.

Как найти площадь неравностороннего прямоугольника

На данной странице калькулятор поможет рассчитать площадь четырехугольника онлайн. Для расчета задайте длину сторон, длины диагоналей и угол между ними, противолежащие углы, радиус окружности.

Четырёхугольник — многоугольник, состоящий из четырех точек (вершин) и четырёх отрезков (сторон), попарно соединяющих эти точки.

Через диагонали и угол между ними

Формула для нахождения площади четырехугольников через диагонали и угол между ними:

Через стороны и противолежащие углы

Формула для нахождения площади четырехугольников через стороны и противолежащие углы:

Площадь вписанного четырехугольника в окружность

Формула Брахмагупты для нахождения площади вписанного четырехугольника в окружность:

Площадь описанного четырехугольника около окружности через радиус

Формула для нахождения площади описанного четырехугольника около окружности через радиус:

Площадь произвольного четырехугольника, формулы и калькулятор для вычисления в режиме онлайн.

Площадь четырехугольника – это численная характеристика, характеризующая размер плоскости, ограниченной геометрической фигурой, образованной четырьмя последовательно соединенными отрезками.

Для вычисления площади произвольного четырехугольника применяются различные формулы, в зависимости от известных исходных данных. Ниже приведены формулы и калькулятор, который поможет вычислить площадь произвольного четырехугольника или проверить уже выполненные вычисления.

В окончании статьи приведены ссылки для вычисления частных случаев четырехугольников: квадрата, трапеции, параллелограмма, прямоугольника, ромба.

Площадь четырехугольника по диагоналям и углу между ними

Площадь четырехугольника через стороны и углы между этими сторонами

При вычислении площади четырехугольника с использованием данной формулы, необходимо предварительно вычислить полупериметр четырехугольника по формуле:

Площадь четырехугольника вписанного в окружность, вычисляемая по Формуле Брахмагупты

Данная формула справедлива только для четырехугольников, вокруг которых можно описать окружность.

При вычислении площади четырехугольника с использованием данной формулы, необходимо предварительно вычислить полупериметр четырехугольника по формуле:

Площадь четырехугольника в который можно вписать окружность

Данная формула справедлива только для четырехугольников, в которые можно вписать окружность. Вписанная окружность должна иметь точки соприкосновения со всеми четырьмя сторонами четырехугольника.

При вычислении площади четырехугольника с использованием данной формулы, необходимо предварительно вычислить полупериметр четырехугольника по формуле:

Площадь четырехугольника в который можно вписать окружность, определяемая через стороны и углы между ними

Данная формула справедлива только для четырехугольников, в которые можно вписать окружность. Вписанная окружность должна иметь точки соприкосновения со всеми четырьмя сторонами четырехугольника.

Определения

Четырехугольник – это геометрическая плоская фигура, образованная четырьмя последовательно соединенными отрезками.

Площадь – это численная характеристика, характеризующая размер плоскости, ограниченной замкнутой геометрической фигурой.

Площадь измеряется в единицах измерения в квадрате: км 2 , м 2 , см 2 , мм 2 и т.д.

Если в исходных данных угол задан в радианах, то для перевода в градусы вы можете воспользоваться нашим «Конвертером величин». Или вычислить самостоятельно по формуле: 1 рад × (180/π) ° = 57,296°

Для вычисления частных случаев четырехугольников можно воспользоваться формулами и калькуляторами, приведенными в других статьях сайта:

Вычисляет площадь неправильного четырехугольника с известными длинами сторон

С завидным упорством некоторые пользователи Planetcalc оставляют запросы на создание калькулятора для расчета площади неправильного четырехугольника, для которого известны только длины сторон. Подумалось, что остановить их можно только написав вот такой шуточный калькулятор. (Нажмите кнопку «Остановить» для определения площади понравившегося Вам четырехугольника с заданными Вами сторонами).

Как вычислить площадь четырехугольника с разными сторонами

На данной странице калькулятор поможет рассчитать площадь четырехугольника онлайн. Для расчета задайте длину сторон, длины диагоналей и угол между ними, противолежащие углы, радиус окружности.

Четырёхугольник — многоугольник, состоящий из четырех точек (вершин) и четырёх отрезков (сторон), попарно соединяющих эти точки.

Через диагонали и угол между ними

Формула для нахождения площади четырехугольников через диагонали и угол между ними:

Через стороны и противолежащие углы

Формула для нахождения площади четырехугольников через стороны и противолежащие углы:

Площадь вписанного четырехугольника в окружность

Формула Брахмагупты для нахождения площади вписанного четырехугольника в окружность:

Площадь описанного четырехугольника около окружности через радиус

Формула для нахождения площади описанного четырехугольника около окружности через радиус:

На данной странице калькулятор поможет рассчитать площадь четырехугольника онлайн. Для расчета задайте длину сторон, длины диагоналей и угол между ними, противолежащие углы, радиус окружности.

Четырёхугольник — многоугольник, состоящий из четырех точек (вершин) и четырёх отрезков (сторон), попарно соединяющих эти точки.

Через диагонали и угол между ними

Формула для нахождения площади четырехугольников через диагонали и угол между ними:

Через стороны и противолежащие углы

Формула для нахождения площади четырехугольников через стороны и противолежащие углы:

Площадь вписанного четырехугольника в окружность

Формула Брахмагупты для нахождения площади вписанного четырехугольника в окружность:

Площадь описанного четырехугольника около окружности через радиус

Формула для нахождения площади описанного четырехугольника около окружности через радиус:

Вычисляет площадь неправильного четырехугольника с известными длинами сторон

С завидным упорством некоторые пользователи Planetcalc оставляют запросы на создание калькулятора для расчета площади неправильного четырехугольника, для которого известны только длины сторон. Подумалось, что остановить их можно только написав вот такой шуточный калькулятор. (Нажмите кнопку «Остановить» для определения площади понравившегося Вам четырехугольника с заданными Вами сторонами).

Площадь Неправильного Многоугольника Формула Онлайн Калькулятор

  • Площадь правильного многоугольника — Онлайн калькуляторы
  • Пятиугольник: площадь и периметр — онлайн расчет, формулы
  • Площадь правильного многоугольника | Формулы и расчеты.
  • Расчет площади неправильного многоугольника онлайн.
  • Используя этот онлайн калькулятор, вы сможете найти площадь правильного многоугольника. Определи площадь многоугольника, если: — у многоугольника 8 сторон и r=16 см (если корня в ответе нет, под знаком корня пиши 1) s=⋅−−−−−√см2 — у многоугольника 9 сторон и r=16 см Площадь, Формулы площади, Формула площади, Формулы площади плоских фигур, Формулы площади треугольника, Формулы площади поверхности объемных тел, формула площади поверхности

    Формула для расчета площади неправильного многоугольника.

    1.8 5 Площадь правильного многоугольника; 1.9 6 Площадь неправильного многоугольника; 2 Формула для расчета площади неправильного многоугольника. 2.1 Площадь многоугольника Калькулятор расчета и периметра правильного многоугольника. Расчет по длине стороны: Многоугольника = ((side)² * N) / (4Tan(π / N)) Площадь неправильного шестиугольника по сторонам онлайн калькулятор S = a b:2 – площадь прямоугольного треугольника . В пятом классе мы познакомились с формулами нахождения площади следующих фигур: — прямоугольника — квадрата — прямоугольного треугольника. Нахождение площади сводится к

    Многокутник: площа і периметр — онлайн калькулятор, формула

    Калькулятор Введіть кількість сторін (кількість кутів) n = Введіть 1 величину . a = R = r = Округлити до знаків після коми. площа S = периметр P = Порядок розрахунку Пов’язані посилання. Перетворення одиниць вимірювання довжи� Основная формула вычисления площади правильного многоугольника, онлайн калькулятор поможет быстро найти площадь зная сторону.

    Многоугольник | Онлайн калькулятор

    Онлайн калькулятор вычисляет число сторон, размер каждого внешнего угла и размер каждого внутреннего угла правильного многоугольника. Зная один из этих параметров можно узнать остальные. Калькулятор для расчета площади. Данный онлайн-калькулятор позволяет рассчитать площадь различных геометрических фигур, таких как: Для удобства расчетов вы можете выбрать единицу измерения (миллиметр, сантиметр.

    Онлайн калькулятор вычисляет площадь и периметр пятиугольника. На сайте имеются необходимые формулы, чертежи и краткое, понятное описание. Наш сайт поможет вам сделать необходимые вычисления быстро и просто. Площадь вычисляется в акрах, квадратных метрах, гектарах. В блок поиска местности можно вводить адрес или координаты местоположения. Пользоваться сервисом просто — сначала нужно найти местность, а затем определять.

    Формула для расчета площади неправильного многоугольника.

    Чтобы рассчитать площадь треугольника, найдите Р, для этого сумму всех сторон поделите на 2. Формула расчета площади неправильного многоугольника В онлайн калькулятор можна вводити числа або дроби. Більш детально читайте в правилах вводу. Формула для обрахунку площі повної поверхні циліндра . S = 2 π R h + 2 π R 2 = 2 π R (R + h) де S — площа, R — радіус циліндра, h — висота цилі Калькулятор схема перетворення квадратів або обчислення площа квадрат, квадратний або квадратний візерунок. Обчислити діаметр кола, вписаного або окресленої по колу квадрата, площа схеми. Розрахунок по периметру к�

    Считаем площадь неправильного 4-х угольника

    Площадь неправильного 4-х угольника с заданными сторонами . Следующий калькулятор был создан по неоднократному запросу создать калькулятор, который бы смог считать площадь неправильного четырехугольника, в котором Онлайн калькулятор периметра эллипса. Как узнать периметр эллипса. Поделиться Что такое имплантация зубов? Можно ли сделать свадебный букет самой? Что мешает на пути к успеху и как с этим бороться. При помощи нашего.

    Как рассчитать площадь неправильного многоугольника с.

    Как рассчитать площадь неправильного многоугольника с разными сторонами Обратиться за помощью к математике: формулы для расчета площади многоугольника или овала будут немного посложнее, но и эту задачу можно. Посчитайте площадь правильного многоугольника на удобном онлайн калькуляторе и посмотрите полное решение, а так же теорию по площади правильного многоугольника.

    Площадь правильного многоугольника, формула Для того чтобы вычислить площадь правильного многоугольника его разбивают на равные треугольники с общей вершиной в центре вписанной окружности. Площадь многоугольника. Площадь геометрической фигуры — это характеристика плоского объекта, которая показывает его размер. Площадь невыпуклых многоугольников находится путем разбиения фигуры на более мелкие сос

    Онлайн калькулятор. Площа чотирикутника

    В онлайн калькулятор можна вводити числа або дроби. Більш детально читайте в правилах вводу чисел. n.b. В онлайн калькуляторі можна використовувати величини в однакових одиницях виміру! Площадь многоугольника — в программу нужно ввести координаты углов многоугольника (Хy) и многоугольник будет нарисован, а его площадь, длина сторон и величины углов посчитаны

    Площадь неправильного пятиугольника онлайн калькулятор.

    Данный онлайн-калькулятор позволяет рассчитать различных геометрических фигур, таких как: Для удобства расчетов вы можете выбрать единицу измерения (миллиметр, сантиметр, метр, километр, фут, ярд, дюйм, миля). Также. Площадь неправильного многоугольника по сторонам онлайн Этот показатель обязательно присутствует в перечне параметров участка, зафиксированных в ЕГРН.

    Формула для расчета площади неправильного многоугольника.

    Площадь неправильного многоугольника онлайн калькулятор. В задачах по геометрии часто требуется вычислить площадь многоугольника. Причем он может иметь довольно разнообразную форму. Площадь неправильного многоугольника онлайн калькулятор. проблемы, ур авнения и форму лы на клик от кнопки. лазерн ой, механической, финансов, нефти и газа, структурных и т. своем кла ссе в школе. каждый месяц. читься. Расчет объемов земляных работ. Треншея. Котлован. Выбран грунт: Объем траншеи (V) = м3 Площадь поперечного сечения (F) = м2 Внимание: если вы задаете вид грунта, то программа сама высчитывает размер a 2 (по коэф. m из таблицы в.

    Площадь многоугольника по формуле Пика — Онлайн калькуляторы

    Площадь многоугольника по формуле Пика. Формула Пи́ка (или теорема Пи́ка) — классический результат комбинаторной геометрии и геометрии чисел, даёт выражение для площади многоугольника с целочисленными вершинами. Формула для расчета радиус круга через диаметр: R=D/2 Где R — радиус круга, S – площадь круга, P – длина круга, D — диаметр, π – число Пи которое всегда примерно равно 3,14.

    Площадь многоугольника Калькулятор считает площадь многоугольника по введенным вами сторонами и диагоналям, главное чтобы диагонали делили многоугольник на несколько треугольников, которые в свою очередь не. Формулы для нахождения площади правильного многоугольника. Удобный калькулятор для расчета площади онлайн. Как рассчитать площадь четырехугольника. На данной странице калькулятор поможет.

    Онлайн калькулятор: Площадь неправильного четырехугольника.

    Онлайн калькулятор. Вычисляет площадь неправильного четырехугольника с известными. Приведены все известные формулы и онлайн-калькулятор для расчета площади поверхности человеческого тела — Мостеллера, Дюбуа и Дюбуа и др. Такого рода расчёты необходимы не только в медицине и фармакологии, но и в. Калькулятор уклонов ; Угол, град. Уклон, % Катет, мм. Угол. Рисунок. Описание. С помощью кальулятора перевода из процентов в градусы можно визуально измерить и оценить необходимый угол (уклон). Данные расчеты очень часто

    Онлайн калькулятор: Площадь многоугольника

    Онлайн калькулятор. Расчет площади многоугольника по введенным сторонами и диагоналям. Площадь многоугольника — скачать Площадь многоугольника 2.8.0, Площадь многоугольника — в программу нужно ввести координаты углов многоугольника (Хy) и многоугольник будет нарисован, а его площадь, длина сторон и. Калькулятор расчета площади неправильного многоугольника формула; Площадь неправильного многоугольника онлайн калькулятор; Калькулятор расчета площади земельного участка неправильной.

    Площадь неправильного многоугольника онлайн калькулятор.

    Онлайн калькулятор. Вычисляет площадь неправильного четырехугольника с известными. Расчет площади многоугольника он-лайн. Калькулятор площади многоугольника по разным формулам. План проведения урока по теме: «Формулы для вычисления площади правильного многоугольника, его стороны и радиуса вписанной окружности» Тема программы: Длина окружности и площадь круга

    Расчет площади многоугольника онлайн

    Калькулятор считает площадь многоугольника по введенным вами сторонами и диагоналям, главное чтобы диагонали делили многоугольник на несколько треугольников, которые в свою очередь не пересекались бы между собой. Онлайн калькулятор. Площадь четырехугольника. Используя этот онлайн калькулятор, вы сможете найти площадь четырехугольника. Воспользовавшись онлайн калькулятором для вычисления площади четырехугольника, вы получ Рассчитать площадь неправильного многоугольника онлайн – Юридические услуги. Правильный пятиугольник – это многоугольник с пятью равными сторонами. Все соседние стороны образуют угол.

    Калькулятор для расчета площади

    Площадь прямоугольника. Онлайн-калькулятор Калькулятор дней Площадь круга. Онлайн-калькулятор Площадь треугольника. Онлайн-калькулятор Нахождение НОД и НОК чисел Содержание Считаем площадь неправильного 4-х угольникаКалькулятор для расчета площадиПлощадь четырехугольника. Пример написания калькулятораПлощадь четырехугольника | Калькулятор онлайн бесплатноПлощадь четыре

    Площадь правильного многоугольника — Геометрия 9 класс.

    Определи площадь многоугольника, если: — у многоугольника 12 сторон и R = 18 см (если корня в ответе нет, под знаком корня пиши 1) S = см 2 — у многоугольника 10 сторон и R = 18 см (ответ округли до целых) S = см 2. спросил 17 Фев, 15 от it В Рассчитать периметр многоугольника онлайн. Пошаговое решение по основной формуле нахождения периметра многоугольника. Справочник Студворк!

    Онлайн калькулятор. Вычисляет площадь неправильного четырехугольника с известными. Онлайн калькулятор. Расчет площади многоугольника по введенным сторонами и диагоналям. Адлер Кирова Частный Сектор. Площадь неправильного многоугольника онлайн калькулятор. В задачах по геометрии часто требуется вычислить площадь многоугольника. Причем он может иметь довольно разнообразную форму. 1.8 5 Площадь правильного многоугольника; 1.9 6 Площадь неправильного многоугольника; 2 Формула для расчета площади неправильного многоугольника. 2.1 Площадь многоугольника Площадь многоугольника Калькулятор считает площадь многоугольника по введенным вами сторонами и диагоналям, главное чтобы диагонали делили многоугольник на несколько треугольников, которые в свою очередь не. Используя этот онлайн калькулятор, вы сможете найти площадь правильного многоугольника. Работа В Липецке Свежие Вакансии Дворник. Чтобы рассчитать площадь треугольника, найдите Р, для этого сумму всех сторон поделите на 2. Формула расчета площади неправильного многоугольника Калькулятор считает площадь многоугольника по введенным вами сторонами и диагоналям, главное чтобы диагонали делили многоугольник на несколько треугольников, которые в свою очередь не пересекались бы между собой. Как В Фотошопе Сделать Абстракцию. Онлайн калькулятор. Вычисляет площадь неправильного четырехугольника с известными. Площадь прямоугольника. Онлайн-калькулятор Калькулятор дней Площадь круга. Онлайн-калькулятор Площадь треугольника. Онлайн-калькулятор Нахождение НОД и НОК чисел Калькулятор Введіть кількість сторін (кількість кутів) n = Введіть 1 величину . a = R = r = Округлити до знаків після коми. площа S = периметр P = Порядок розрахунку Пов’язані посилання. Перетворення одиниць вимірювання довжи�

  • Как Варит Черный Рис В Мультиварке
  • Каско Страховые Случаи Примеры
  • Какие Витамины Помогают Не Болеть Детям
  • Что Дешевле Построить Дом Кирпичный Или Деревянный
  • Вакансии Уборщицы На Неполный Рабочий День В Чите
  • Диски На Уаз Максимальный Размер
  • Чем Заглушить Зубную Боль Дома
  • Почему Мужчина Уходит Молча
  • Как Здравствуйте На Армянском Языке
  • Утеплитель Кнауф Для Деревянного Дома
  • К Чему Сниться Смотреть На Окна Любимого
  • Как Избавиться От Запаха Ремонта В Квартире
  • Где Осаго Дешевле В Твери
  • Наука Английский Язык Урок
  • Новые Коттеджи С Отделкой Под Ключ
  • Кроссовер Через Нижний Блок
  • Цена Страховки На Автомобиль Согаз
  • Бизнес План Диско Бар
  • Как Обновить Вирусные Базы Доктора Веб
  • Водитель Вакансии Спб Частичная Занятость
  • Значения Коэффициентов При Расчете Осаго
  • Фото Авто Тюнинга Ваз
  • Картинки По Английскому Языку Для Детей Местоимения
  • Вакансия Машиниста Компрессорной Установки На Севере
  • Ловля Окуней На Офсетный Крючок Видео
  • Мультфильмы По Басни Крылова
  • Санатория Ассы Какие Лечение
  • На Каких Неделях Проявляется Внематочная Беременность
  • Вакансия Менеджер В Калининском Районе
  • Электрики В Алматы Недорого
  • Если Безработная Какое Пособие По Родам И Беременности
  • Смотреть Мелодраму Про Бизнес Леди
  • С Какого Срока Беременности Увеличивается Аппетит
  • Парикмахер В Щелково Вакансии
  • Принцип Заде Если А И В
  • Read More

    Как рассчитать площадь земельного участка неправильной формы?

    Площадь – важнейшая характеристика земельного участка. Этот показатель необходим при уплате земельного налога или совершении какой-либо сделки с землей.

    СодержаниеПоказать

    Расчет площади участка

    Поскольку площадь является одной из основных характеристик участка, то ее величина приводится в составе данных регистрируемого в ЕГРН участка. Эта площадь вычисляется по сложным формулам кадастровым инженером в процессе проведения межевания. Однако в некоторых случаях желательно оценить площадь участка на местности.

    Для определения площади участка, имеющего простую форму в виде геометрической фигуры типа треугольника, квадрата или прямоугольника, можно использовать простейшие формулы аналитической геометрии. Порядок определения площади участка, имеющего форму неправильного четырехугольника, зависит от типа такого многоугольника.

    На местности наиболее просто произвести замеры длин границ участка с помощью рулетки. Вычислить площадь участка при этом можно вычислить по формуле Брахмагупты:

    S=√(p-a)*(p-b)*(p-c)*(p-d),

    где а, b, c и d – длины границ участка, а p=0,5*(a+b+c+d) – половина его периметра.

    Но такая формула верна только в том частном случае, когда фигура, отображающая форму участка (четырехугольник) может быть вписана в окружность. А это означает, что суммы противоположных углов четырехугольника должны быть равны между собой и равны 180°.

    В других случаях для определения площади участка четырехугольной формы необходимо измерять и углы.

    Например, если известны длины диагоналей фигуры (l1 и l2) и угол между ними φ, то площадь участка равна:

    S=0,5* l1* l2*sin φ.

    Если условие применения формулы Брахмагупты не выполняется (четырехугольник нельзя вписать в окружность), то используется более общий вид формулы:

    S=√(p-a)*(p-b)*(p-c)*(p-d) –a*b*c*d*cos2(α+β)/2,

    где α и β – противолежащие углы.

    Из приведенных формул видно, что информации о четырех измеренных границах участка в форме четырехугольника для вычисления его площади недостаточно. В любом случае необходимо оценить величины противоположных углов.

    Если можно вписать в четырехугольник окружность с радиусом r, то площадь участка будет равна:

    S=p*r.

    Вариант оценки

    Вариантом определения площади участка четырехугольной формы с помощью измерений одной рулеткой является разбиение четырехугольника на 2 треугольника. После измерения сторон участка (a, b, c, d) и одной из диагоналей (l1), площади треугольников S1 и S2 можно определить по формуле Герона:

    S1=√(p1-a)*(p1-d)*(p1- l1),

    где p1=0,5*(a+d+l1) – половина периметра первого треугольника.

    S2=√(p2-b)*(p2-c)*(p2- l1),

    где p2=0,5*(b+c+l1) – половина периметра второго треугольника.

    Тогда площадь всего участка будет равна площадей треугольников:

    S= S1+ S2.

    Выводы

    1. Определить площадь участка четырехугольной неправильной формы только по длинам его границы нельзя. Необходима информация об углах четырехугольника.
    2. Для оценки площади участка такой формы можно использовать вариант с разбиением его на две фигуры треугольной формы. При этом дополнительно к измерениям сторон участка необходимо измерить одну из диагоналей, соединяющих противоположные опорные точки участка.

     

    Формулы площадей всех основных фигур

    1. Формула площади равнобедренной трапеции через стороны и угол

    b — верхнее основание

    a — нижнее основание

    c — равные боковые стороны

    α — угол при нижнем основании

     

    Формула площади равнобедренной трапеции через стороны, (S):

     

    Формула площади равнобедренной трапеции через стороны и угол, (S):

     

     

    2. Формула площади равнобокой трапеции через радиус вписанной окружности

    R — радиус вписанной окружности

    D — диаметр вписанной окружности

    O — центр вписанной окружности

    H — высота трапеции

    α, β — углы трапеции

     

    Формула площади равнобокой трапеции через радиус вписанной окружности, (S):

    СПРАВЕДЛИВО, для вписанной окружности в равнобокую трапецию:

     

     

    3. Формула площади равнобедренной трапеции через диагонали и угол между ними

    d — диагональ трапеции

    α, β — углы между диагоналями

     

    Формула площади равнобедренной трапеции через диагонали и угол между ними, (S):

     

     

    4. Формула площади равнобедренной трапеции через среднюю линию, боковую сторону и угол при основании

    m — средняя линия трапеции

    c — боковая сторона

    α, β — углы при основании

     

    Формула площади равнобедренной трапеции через среднюю линию, боковую сторону и угол при основании, (S ):

     

     

    5. Формула площади равнобедренной трапеции через основания и высоту

    b — верхнее основание

    a — нижнее основание

    h — высота трапеции

     

    Формула площади равнобедренной трапеции через основания и высоту, (S):

     

    Как посчитать площадь неровного прямоугольника

    На данной странице калькулятор поможет рассчитать площадь четырехугольника онлайн. Для расчета задайте длину сторон, длины диагоналей и угол между ними, противолежащие углы, радиус окружности.

    Четырёхугольник — многоугольник, состоящий из четырех точек (вершин) и четырёх отрезков (сторон), попарно соединяющих эти точки.

    Через диагонали и угол между ними

    Формула для нахождения площади четырехугольников через диагонали и угол между ними:

    Через стороны и противолежащие углы

    Формула для нахождения площади четырехугольников через стороны и противолежащие углы:

    Площадь вписанного четырехугольника в окружность

    Формула Брахмагупты для нахождения площади вписанного четырехугольника в окружность:

    Площадь описанного четырехугольника около окружности через радиус

    Формула для нахождения площади описанного четырехугольника около окружности через радиус:

    Вычисляет площадь неправильного четырехугольника с известными длинами сторон

    С завидным упорством некоторые пользователи Planetcalc оставляют запросы на создание калькулятора для расчета площади неправильного четырехугольника, для которого известны только длины сторон. Подумалось, что остановить их можно только написав вот такой шуточный калькулятор. (Нажмите кнопку «Остановить» для определения площади понравившегося Вам четырехугольника с заданными Вами сторонами).

    Инструкция для калькулятора расчета площади земельного участка неправильной формы

    Примите к сведению, калькулятор Вариант 1 считает площадь участка, который вписывается углами в окружность. Проверить подходит ли Ваш участок под это условие можно, учитывая, что противоположные углы вписанного четырехугольника в сумме дают 180°. Если Ваш участок не вписывается в окружность, то калькулятор будет выдавать небольшую погрешность (величина который будет зависеть от того, насколько сильно Ваш участок не вписывается в окружность).

    Впишите размеры сторон AB, BC, CD, DA в метрах.

    Замеры участка проводят, с помощью длинной рулетки, лазерного дальномера, мерного колеса. Также можно изготовить деревянную треногу (наподобие циркуля) зафиксировав расстоянием между «ногами» в 1 м и, шагая таким приспособлением вдоль границы делянки подсчитать количество шагов.

    Нажмите «Рассчитать».

    Онлайн калькулятор позволяет определить площадь (в квадратных метрах, сотках, акрах и гектарах) земельного участка или поля, имеющего неправильную форму. Это пригодится для корректного оформления документов права собственности на землю, продажи, сдачи в аренду или его деления (т.н. межевания) без применения топографической съемки и специальных картографических программ. Также иногда просто нужно посчитать сколько соток имеет участок, чтобы прикинуть его рыночную стоимость.

    195

    Площадь любого неправильного четырехугольника

    Плоская фигура, ограниченная четырьмя отрезками прямых, называется неправильным четырехугольником. Площадь любого неправильного четырехугольника можно вычислить, разделив его на треугольники.

    Пример :

    Найдите площадь четырехугольника $$ ABCD $$, стороны которого равны $$ 9 $$ м, $$ 40 $$ м, $$ 28 $$ м и $$ 15 $$ м соответственно, а угол между первыми двумя сторонами является прямым. угол.2}} = \ sqrt {1681} = 41 \\ \ end {собрано} \]

    Теперь площадь $$ \ Delta ABD = \ frac {1} {2} \ times 40 \ times 9 = 180 $$ m
    In $$ \ Delta BCD $$, $$ BD = a = 41 $$ m, $$ DC = b = 28 $$ m, $$ CB = c = 15 $$ m
    $$ \ поэтому s = \ frac {{a + b + c}} {2} = \ frac {{41 + 28 + 15}} {2} = 42 $$ m

    Теперь
    \ [\ begin {gather} {\ text {Area}} \, {\ text {of}} \, \ Delta BCD = \ sqrt {s (s — a) (s — b) (s — c)} \, \\ {\ text {Area}} \, {\ text {of}} \, \ Delta BCD = \ sqrt {42 (42 — 41) (42 — 28) (42 — 15)} = \ sqrt {42 \ times 14 \ times 27} = 126 \, sq \, m \\ \ end {собрано} \]

    Площадь четырехугольника $$ ABCD $$$$ = $$ Площадь $$ \ Delta ABD $$ $$ + $$ Площадь $$ \ Delta BCD $$
    Площадь четырехугольника $$ ABCD $$ $$ = (180 + 126) = 306 $$ квадратных метров.

    Пример :

    В четырехугольнике диагональ составляет $$ 42 $$ см, а два перпендикуляра на нем от других вершин — $$ 8 $$ см и $$ 9 $$ см соответственно. Найдите площадь четырехугольника.

    Решение :

    Учитывая, что из рисунка $$ AC = 42 $$ м, $$ BN = 9 $$ м, $$ DM = 8 $$ м
    Площадь $$ ABCD = $$ Площадь $$ \ Delta ABC + $$ область $$ \ Delta ACD $$
    Площадь $$ ABCD $$$$ = \ frac {1} {2} \ times 9 \ times 42 + \ frac {1} {2} \ times 8 \ умножить на 42 = 189 + 168 = 357 $$ кв.

    неправильные четырехугольники | Площадь, определение и видео // Tutors.com

    Содержание

    Иногда жизнь проста и понятна. Квадраты привычны и удобны, регулярны и предсказуемы. С другой стороны, прямоугольники, трапеции, воздушные змеи и другие необычные четырехугольники не так просты. Для неправильных четырехугольников даже такая простая задача, как определение их площади, может стать проблемой.

    1. Четырехугольник Определение
    2. Правильный четырехугольник
    3. Неправильные четырехугольники
    4. Площадь неправильных четырехугольников
    5. Площадь очень неправильных четырехугольников

    Четырехугольник

    Напомним, четырехугольник (латинское: «четыре стороны» ) — это двухмерная плоская фигура, которая использует четыре отрезка линии, чтобы ограничить пространство.Поскольку определение очень широкое, многие общие формы представляют собой четырехугольники:

    1. Квадраты
    2. Прямоугольники
    3. Трапеции
    4. Параллелограммы
    5. Воздушные змеи
    6. Ромбы

    Правильный четырехугольник

    Из длинного списка только квадрат является правильным четырехугольником . У правильных многоугольников совпадающие стороны и углы. Вы легко можете видеть, что прямоугольник может иметь четыре внутренних угла 90 °, но у него не обязательно должны быть четыре стороны равной длины. 2.

    Если у вас есть квадрат со сторонами 17 см, его площадь составляет 289 см2. Если у вас есть прямоугольник с двумя сторонами 17 см и двумя сторонами 34 см, площадь будет 17 x 34 = 578 квадратных см.

    Вычислить площадь для большинства других неправильных четырехугольников может быть непросто. Площадь параллелограмма или ромба, например, равна его высоте (или высоте), а не длине его короткой стороны, умноженной на его основание. Для трапеции вы должны найти среднее значение двух оснований и умножить это значение на высоту трапеции.

    Воздушный змей , который имеет две смежные короткие стороны и две смежные длинные стороны, имеет формулу площади, основанную на его диагоналях, d1 и d2:

    Площадь очень

    Неровные Четырёхугольники

    Наш предыдущий пример неправильного четырехугольника, MATH, показывает, что четыре стороны не гарантируют симметричную форму. Чтобы найти площадь таких неправильных четырехугольников, следуйте трехэтапной стратегии:

    1. Разделите четырехугольник на два треугольника, построив диагональ, не нарушающую известный внутренний угол
    2. Рассчитайте площадь каждого треугольника по формуле
    3. Добавьте площади двух треугольников

    Для нашего четырехугольника MATH соединение вершины A с вершиной H разбивает фигуру на △ MAH и △ ATH.Вы не знаете высоты h ни одного из треугольников, поэтому вы не можете рассчитать площадь, используя 1 / 2bh.

    Вместо этого проявите немного творчества (математика полна творчества), основывая один факт на другом. В нашем четырехугольнике MATH, если мы знаем один угол, мы можем использовать эти четыре шага, чтобы найти общую площадь:

    1. Зная, что включенный угол T составляет 120 °, вы можете использовать Side-Angle-Side, чтобы найти площадь △ ATH
    2. Зная площадь △ ATH, вы можете использовать закон косинусов для вычисления неизвестной длины диагонали AH
    3. Зная длину диагонали AH, вы можете использовать формулу Герона для вычисления площади △ MAH
    4. Зная площади двух треугольников, сложите их, чтобы получить площадь неправильного четырехугольника

    Обратите внимание, что вы должны работать последовательно, и для начала у вас должны быть некоторые основные факты.2 = 189

  • t ≈ 13,747 см
  • Теперь у нас есть приблизительная длина стороны AH, равная 13,747 см, поэтому мы можем использовать Формулу Герона , чтобы вычислить площадь другой части нашего четырехугольника.

    Используйте формулу Герона

    Формула Герона зависит от знания полупериметра или половины периметра треугольника. Для нашего △ MAH размер трех сторон:

    1. MA = 7 см
    2. AH = 13,747 см
    3. HM = 14 см

    Полупериметр s , это половина суммы сторон:

    С точностью до тысячных квадратного сантиметра мы имеем площадь четырехугольника MATH!

    Краткое содержание урока

    Теперь, когда вы полностью изучили урок, вы можете определять четырехугольники, различать правильные и неправильные четырехугольники, а также вспоминать и объяснять отличительные свойства правильных и неправильных четырехугольников. Теперь вы также можете применить формулу длина-ширина в качестве формулы для вычисления площади правильных и некоторых неправильных четырехугольников, и, когда формула длины-ширины не может применяться, найти площадь неправильных четырехугольников, используя другие стратегии, в том числе с помощью закона косинусов и формулы Герона.

    Следующий урок:

    Что такое четырехугольник?

    Площадь неправильного многоугольника

    Площадь неправильного многоугольника — Math Open Reference

    В отличие от обычного многоугольника, если вы не знаете координаты вершин, нет простой формулы для площади неправильного многоугольника.
    Каждая сторона могла быть разной длины, и каждый внутренний угол мог быть разным. Он также может быть выпуклым или вогнутым.

    Если вы знаете координаты вершин многоугольника, есть два метода:

    1. Ручной метод.См. Площадь многоугольника (Координатная геометрия).
    2. Компьютер
      алгоритм.
      См. Алгоритм определения площади любого многоугольника.
    Итак, как это сделать?

    Один из подходов — разбить фигуру на части, которые может решить — обычно треугольники,
    так как есть много способов вычислить площадь треугольников.
    Как именно вы это сделаете, зависит от того, что вам дано для начала.
    Поскольку это очень вариативно, нет простого правила, как это сделать.
    В приведенных ниже примерах представлены несколько основных подходов, которые можно попробовать.

    1. Разбейте на треугольники и прибавьте

    .

    На рисунке выше многоугольник можно разбить на треугольники, нарисовав все диагонали.
    от одной из вершин. Если вы знаете достаточно сторон и углов, чтобы определить площадь каждого из них, вы можете просто сложить их, чтобы найти общую сумму.
    Не бойтесь рисовать где-нибудь лишние линии, если они помогут найти фигуры, которые вы сможете решить.

    Здесь неправильный шестиугольник разделен на 4 треугольника добавлением красных линий.
    (См. Площадь треугольника)

    2.Найдите «недостающие» треугольники, затем вычтите

    На рисунке выше общая форма представляет собой правильный шестиугольник, но отсутствует треугольная деталь.

    Мы знаем, как найти площадь правильного многоугольника, поэтому мы просто вычитаем площадь «недостающего» треугольника, созданного путем рисования красной линии.
    (См. Площадь правильного многоугольника и
    Площадь треугольника.)

    3. Рассмотрим другие формы

    На рисунке выше фигура представляет собой неправильный шестиугольник, но его симметрия позволяет разбить его на два параллелограмма.
    нарисовав красную пунктирную линию.(при условии, конечно, что линии, которые выглядят параллельными, действительно таковы!)

    Мы знаем, как найти площадь параллелограмма, поэтому мы просто находим площадь каждого из них и складываем их вместе.
    (См. Площадь параллелограмма).

    Как видите, существует бесконечное количество способов разбить фигуру на части, которыми легче управлять.
    Затем вы складываете или вычитаете площади частей.
    То, как вы это делаете, зависит от личных предпочтений и того, что вам дают для начала.

    4.Если известны координаты вершин

    Если вам известны координаты x, y вершин (углов) фигуры, есть способ найти область напрямую.
    См. Площадь многоугольника (Координатная геометрия). Это работает для всех типов многоугольников (правильных, неправильных, выпуклых, вогнутых). Также есть компьютер
    алгоритм
    это делает то же самое. См. Алгоритм определения площади любого многоугольника.

    Другие темы многоугольников

    Общий
    Типы многоугольника
    Площадь различных типов полигонов
    Периметр различных типов многоугольников
    Углы, связанные с многоугольниками
    Именованные многоугольники

    (C) Открытый справочник по математике, 2011 г.
    Все права защищены.

    Площадь вписанного четырехугольника

    Площадь вписанного четырехугольника — Math Open Reference

    Формула для вычисления площади вписанного или вписанного четырехугольника

    , если вам известны длины сторон (a, b, c, d).

    Попробуйте это Перетащите любую оранжевую точку. Обратите внимание на изменение формулы для расчета площади.

    Напомним, что
    вписанный (или «циклический») четырехугольник — это тот, в котором четыре
    все вершины лежат на окружности.Используя формулу ниже, вы можете рассчитать площадь четырехугольника.

    где a, b, c, d — длины сторон, а p — половина периметра:

    На рисунке выше перетащите любую вершину по кругу. Обратите внимание, как рассчитываются полупериметр (p) и площадь.

    ‘Скрещенные’ полигоны

    На рисунке выше, если вы перетащите точку мимо соседа, четырехугольник станет «пересеченным», когда одна сторона пересекается с другой.
    В таких «скрещенных» четырехугольниках формула площади больше не выполняется.(Большинство свойств многоугольников недействительны при пересечении многоугольника).

    Сходство с формулой Герона

    Напомним, что
    Формула Герона для площади треугольника:

    где p — половина периметра, как здесь.

    Эти две формулы очень похожи. Если взять формулу Брахмагупты и установить d (длину четвертой стороны) равным нулю,
    четырехугольник становится треугольником. В формуле Брахмагупты термин (p-d) становится просто p, и формулы также остаются такими же.

    На рисунке выше, если вы будете осторожны, вы можете перетащить точку D, чтобы она оказалась поверх A, сделав d равным нулю, что иллюстрирует это сходство.

    Из этого вы можете видеть, что формула Герона — это просто частный случай формулы Брахмагупты. Напомним также, что все треугольники циклические.
    То есть вы всегда можете нарисовать круг через три вершины. Видеть
    Окружность треугольника.

    Что попробовать

    На рисунке выше

    1. Щелкните «Скрыть подробности».
    2. Перетащите вершины, чтобы создать новый (не скрещенный) четырехугольник.
    3. Вычислите площадь четырехугольника по формуле Брахмагупты.
    4. Оцените площадь, считая квадраты. Каждая — одна квадратная единица.
    5. Нажмите «показать подробности», чтобы проверить свой ответ.

    Другие темы многоугольников

    Общий
    Типы многоугольника
    Площадь различных типов полигонов
    Периметр различных типов многоугольников
    Углы, связанные с многоугольниками
    Именованные многоугольники

    (C) Открытый справочник по математике, 2011 г.
    Все права защищены.

    Что такое площадь четырехугольника?

    Что такое площадь четырехугольника?

    Четырехугольник — это многоугольник, который получается соединением четырех вершин, у него четыре стороны и четыре угла. Есть два типа четырехугольников — правильные и неправильные четырехугольники. Некоторые примеры четырехугольников: квадрат, прямоугольник, ромб, трапеция и параллелограмм.

    Площадь многоугольника — это пространство, занимаемое плоской формой.Это сумма площадей правильных и неправильных треугольников внутри.

    Измерение площади четырехугольника

    Чтобы оценить площадь четырехугольника, мы разделим его на две основные геометрические фигуры, например, треугольники. Затем мы находим площадь двух отдельных треугольников, используя формулу, и складываем эти площади, чтобы найти площадь четырехугольника.

    Расчет площади четырехугольника

    (B и D) по диагонали переменного тока.

    Площадь четырехугольника ABCD = Площадь ABC + Площадь △ ADC

    Итак, площадь четырехугольника ABCD = (½ × AC × BE) + (½ × AC × DF)

    Мы можем вычислить площади четырехугольников различных типов по данной формуле. Для четырехугольника ABCD, если мы используем сантиметр в качестве единицы измерения, единицей измерения площади будет см 2 .

    Площадь параллелограмма

    Чтобы оценить площадь параллелограмма, нарисуйте перпендикуляр от одной из вершин к основанию.Этот перпендикуляр и есть высота. Таким образом, площадь будет произведением базы и высоты.

    Площадь параллелограмма = основание x высота

    Площадь = 12 × 6 = 72 см

    Площадь ромба

    Чтобы найти площадь ромба, разделим четырехугольник на два равнобедренных треугольника, используя две диагонали. В данном ромбе ABCD точка пересечения этих диагоналей равна E. Таким образом, площадь ромба равна:

    Площадь ромба ABCD = Площадь ABC + Площадь △ ADC

    ⟹ Площадь ромба ABCD = (½ x AC x BE) + (½ x AC x ED)

    ⟹ Площадь ромба ABCD = ½ x AC (BE + ED)

    ⟹ Площадь ромба ABCD = ½ x AC x BD

    Площадь квадрата

    Используя это соотношение, мы также можем найти площадь квадрата ABCD

    Площадь квадрата ABCD = Площадь ABC + Площадь △ BCD

    ⟹ Площадь △ ABC = ½ * AC * AB

    ⟹ Площадь △ ABC = ½ * AC * AC (как AC = AB)

    ⟹ Площадь △ ABC = ½ * AC2

    Аналогично, площадь △ BCD = ½ * CD2

    Так как AC = CD, площадь △ BCD будет ½ * AC2

    Таким образом, площадь квадрата ABCD = 2 * (½ * AC2) = AC2

    Следовательно, Площадь квадрата ABCD — это квадрат стороны.

    Площадь прямоугольника

    Площадь прямоугольника по приведенной выше формуле будет равна произведению двух его смежных сторон, основания и высоты. Мы представляем это как:

    Заявка

    Реальное применение четырехугольника и его площади очень полезно в области дизайна, сельского хозяйства и архитектуры. Эта концепция очень полезна при расширенном проектировании навигационных карт, масштабируемых с точностью до фактических расстояний и площадей.

    Площадь, покрытая четырехугольником, образованным соединением четырех разных мест на карте

    Интересные факты

    • Термин четырехугольник — это комбинация слов Quadri + Lateral, что означает «четыре стороны».

    • За исключением квадрата, все четырехугольники неправильные. Они также известны как «Четырехугольник» и «Тетрагон» (четыре и многоугольник).

    • Сумма всех углов внутри четырехугольника всегда равна 360 °.

    Сопутствующая математическая лексика

    Неправильные четырехугольники: определение и площадь — математический класс [видео 2021 года]

    Площадь неправильного четырехугольника

    Предположим, мы хотим разместить стеклянную панель в окне, но нам нужно определить ее размер. Другими словами, мы хотим знать площадь окна. В данном случае мы имеем дело с прямоугольником длиной 5 футов и шириной 2 фута.У нас есть хорошо известная формула для вычисления площади прямоугольника: длина, умноженная на ширину. Следовательно, мы находим площадь нашего окна, умножая 5 футов на 2 фута, чтобы получить 10 квадратных футов.

    • Площадь = длина × ширина = 5 × 2 = 10

    Это размер стеклянной панели, которая нам нужна для окна.

    Это был действительно простой процесс. Это здорово, когда мы работаем с неправильным четырехугольником, который имеет красивую формулу площади, как это было у прямоугольного окна.

    Однако, поскольку все неправильные четырехугольники разные, у нас нет хорошей универсальной формулы, которую мы могли бы использовать для всех из них. Когда дело доходит до определения площади неправильного четырехугольника, для которого нет известной формулы площади, хорошей стратегией является использование следующих шагов:

    1. Разделите четырехугольник на два треугольника, нарисовав диагональ.
    2. Используйте различные формулы и свойства, чтобы найти площадь каждого из треугольников.
    3. Сложите площади треугольников.

    Звучит достаточно просто, но это может быть немного сложно. Давайте рассмотрим пример этого.

    Пример неправильного четырехугольника

    Предположим, мы хотим найти площадь показанного неправильного четырехугольника.

    Хммм… ну, первый шаг в нашей стратегии достаточно прост. Мы просто разбиваем четырехугольник на два треугольника, рисуя диагональ.

    Теперь все становится сложнее. Мы не можем использовать формулу для площади треугольника (1/2) (основание) (высота), так как мы не знаем высоты треугольников. Как говорится на втором этапе нашей стратегии, нам нужно будет использовать различные правила и свойства, чтобы найти их области. В данном случае это следующие:

    Метод SAS : Площадь треугольника с двумя соседними сторонами длиной a и b , а угол θ, являющийся углом между этими сторонами, равен 1/2 ab грех (θ).

    Закон косинусов : В треугольнике со сторонами a , b и c и углами A , B и C , противоположными их сторонам, мы имеем следующие отношения:

    • c 2 = a 2 + b 2 — 2 ab cos ( C )
    • b 2 = a 2 + c 2 — 2 ac cos ( B )
    • a 2 = b 2 + c 2 — 2 до н.э. cos ( A )

    Формула Герона : Если треугольник имеет длину стороны a , b и c , то его площадь равна:

    • Площадь = √ ( s ( s a ) ( s b ) ( s c )), где s = ( a + b + c ) / 2.

    Я знаю, что это выглядит много, но это просто показывает, насколько это может быть связано.

    Обратите внимание, что в нашем неправильном четырехугольнике мы можем найти площадь треугольника ABC , используя метод SAS: a = 3, b = 12 и θ = 120 градусов.

    • 1/2 ab sin (θ) = (1/2) (3) (12) sin (120) ≈ 15,6

    Отлично! У нас есть площадь одного из наших треугольников.

    Если мы сможем найти длину диагонали, у нас будут все стороны треугольника ACD , поэтому мы можем использовать формулу Герона, чтобы найти площадь.Чтобы найти длину диагонали, мы можем использовать закон косинусов с a = 3, b = 12 и C = 120, а затем решить относительно c .

    Получаем, что длина диагонали примерно 13,75 единицы. Теперь мы просто используем формулу Герона с a = 7, b = 14 и c = 13,75. Сначала находим s .

    • с = ( a + b + c ) / 2 = (7 + 14 + 13.75) / 2 = 17,375

    Теперь мы используем формулу, чтобы найти площадь треугольника ACD :

    Получаем, что площадь треугольника ACD составляет примерно 47 квадратных единиц. Наконец, мы просто складываем площади двух треугольников:

    • Площадь четырехугольника ABCD = 15,6 + 47 = 62,6

    Та-да! Площадь неправильного четырехугольника составляет 62,6 квадратных единицы. Уф! Это был процесс, но мы его сделали!

    Резюме урока

    Давайте уделим пару минут тому, чтобы повторить, что мы узнали о четырехугольниках, как правильных, так и неправильных, и о том, как определить площадь неправильных четырехугольников.

    Четырехугольник имеет четырехугольную и четырехугольную форму.

    Правильный четырехугольник — четырехугольник, все стороны которого имеют одинаковую длину.

    Неправильный четырехугольник — это четырехугольник, который не является правильным, поэтому все его стороны не имеют одинаковой длины.

    У некоторых неправильных четырехугольников, таких как воздушный змей или прямоугольник, есть красивые формулы для их площадей. Однако у некоторых неправильных четырехугольников нет формулы площади.В этом случае хорошая стратегия поиска области выглядит следующим образом:

    1. Разделите четырехугольник на два треугольника.
    2. Используйте формулы и свойства, чтобы найти площади каждого треугольника.
    3. Сложите площади треугольников.

    Шаг второй является самым сложным, потому что существует ряд различных сценариев, и каждый из них требует различных формул и свойств для определения площадей треугольников. Чем больше мы тренируемся с этими типами четырехугольников, тем легче это становится, так что продолжайте практиковаться!

    Как рассчитать площадь неправильной формы

    Обновлено 17 ноября 2020 г.

    Лиза Мэлони

    Когда вы впервые начинаете вычислять площадь, вы получаете простые формы с четко определенными формулами для определения площади: круги, треугольники, квадраты и прямоугольники, например.Но что происходит, когда вы сталкиваетесь с формой, которая не вписывается в эти категории? Пока вы не войдете в дивный новый мир математических интегралов, лучший способ найти области неправильных форм — это разделить их на формы, с которыми вы уже знакомы.

    TL; DR (слишком долго; не читал)

    Самый простой способ вычислить площадь неправильной формы — разделить ее на знакомые формы, вычислить площадь знакомых форм, а затем суммировать эти вычисления площади, чтобы получить площадь неправильной формы, которую они составляют.

      Соберите формулы площади для форм, с которыми вы уже знакомы. Наиболее распространенные формы и их формулы включают:

      \ text {Площадь квадрата или прямоугольника} = l × w

      , где l — длина, а w — ширина.

      \ text {Площадь треугольника} = \ frac {1} {2} (b × h)

      , где b — основание треугольника, а h — его высота по вертикали.

      \ text {Площадь параллелограмма} = b × h

      , где b — основание параллелограмма, а h — его высота по вертикали.2

      , где r — радиус окружности.

      Используйте свое воображение, чтобы разделить имеющуюся неправильную форму на более знакомые формы. Иногда рисование формы с последующим добавлением линий для подразделений помогает визуализировать ее и отслеживать соответствующие измерения для каждого измерения. Например, представьте, что вам нужно найти область пятиугольной формы, которая не является шестиугольником, но имеет три перпендикулярные стороны, противоположные «точке». Немного подумав, вы можете разделить это на прямоугольник, который стыкуется с треугольником, причем треугольник образует «точку» формы.

      Вернитесь к формулам площади, чтобы узнать размеры, которые вам понадобятся для расчета площади каждой разделенной фигуры. В этом случае вам понадобятся базовая и вертикальная высота треугольника, а также длина и ширина (или две смежные стороны) прямоугольника. Если вы решаете математическую задачу в школе, вы, вероятно, получите по крайней мере некоторые из этих измерений и, возможно, потребуется использовать некоторую базовую алгебру или геометрию, чтобы найти любые недостающие измерения. Если вы работаете в реальном мире, возможно, вы сможете заполнить некоторые измерения путем физического измерения.

    Формулы площадей всех основных фигур

    1. Формула площади равнобедренной трапеции через стороны и угол

    b — верхнее основание

    a — нижнее основание

    c — равные боковые стороны

    α — угол при нижнем основании

     

    Формула площади равнобедренной трапеции через стороны, (S):

     

    Формула площади равнобедренной трапеции через стороны и угол, (S):

     

     

    2. Формула площади равнобокой трапеции через радиус вписанной окружности

    R — радиус вписанной окружности

    D — диаметр вписанной окружности

    O — центр вписанной окружности

    H — высота трапеции

    α, β — углы трапеции

     

    Формула площади равнобокой трапеции через радиус вписанной окружности, (S):

    СПРАВЕДЛИВО, для вписанной окружности в равнобокую трапецию:

     

     

    3. Формула площади равнобедренной трапеции через диагонали и угол между ними

    d — диагональ трапеции

    α, β — углы между диагоналями

     

    Формула площади равнобедренной трапеции через диагонали и угол между ними, (S):

     

     

    4. Формула площади равнобедренной трапеции через среднюю линию, боковую сторону и угол при основании

    m — средняя линия трапеции

    c — боковая сторона

    α, β — углы при основании

     

    Формула площади равнобедренной трапеции через среднюю линию, боковую сторону и угол при основании, (S ):

     

     

    5. Формула площади равнобедренной трапеции через основания и высоту

    b — верхнее основание

    a — нижнее основание

    h — высота трапеции

     

    Формула площади равнобедренной трапеции через основания и высоту, (S):

     

    Площадь неправильных форм — расчет, примеры и часто задаваемые вопросы

    Неправильные формы — это многоугольники с пятью или более сторонами различной длины. Эти формы или фигуры можно дополнительно разложить на известные формы, такие как треугольники, квадраты и четырехугольники, чтобы оценить площадь.

    Вот некоторые примеры неправильной формы:

    Объекты повседневной жизни неправильной формы

    Как рассчитать площадь неправильной формы?

    Определение площади неправильной формы

    Существуют различные методы оценки площади неправильной формы:

    • Оценка площади с использованием единичных квадратов.

    • Разделение неправильной формы на две или более правильных формы.

    • Разделение неправильной формы кривыми на две или более правильных формы.

    Как найти область неровной формы?

    Оценка площади с помощью единичных квадратов

    Мы можем использовать этот метод для фигур с кривыми, отличными от идеального круга или полукругов и неправильных четырехугольников. В этом методе мы сначала делим фигуру на единичные квадраты. Общее количество единичных квадратов, попадающих в форму, используется для определения общей площади.

    Например: рассчитайте площадь, посчитав единичные квадраты на приведенном ниже рисунке.

    Ответ: Если обозначить каждую единицу квадрата в сантиметрах, площадь будет равна 6 см 2 .

    Расчет площади неправильной формы с изогнутыми краями

    Разделение неправильной формы на две или более правильных формы

    Используйте этот метод для расчета площади неправильной формы, которая представляет собой комбинацию треугольников и многоугольников.Используя предопределенные формулы, вычислите площадь таких фигур и сложите их, чтобы получить общую площадь.

    Например, в приведенной ниже неправильной форме мы разделим несколько ребер на треугольник и три многоугольника.

    Общую площадь фигуры можно рассчитать, добавив отдельную площадь:

    Общая площадь = Площадь (ABIM) + Площадь (BCGH) + Площадь (CDEF) + Площадь (JKL)

    ⇒ Общая площадь = (AB × BI) + (BC × CG) + (CD × DE) + (12 × LJ × KO)

    ⇒ Общая площадь = (10 × 5) + (3 × 3) + (2 × 2) + (1⁄ 2 × 4 × 4)

    ⇒ Общая площадь = 50 + 9 + 4 + 8

    ⇒ Общая площадь = 71 см 2

    Расчет площади неправильной формы

    Для расчета площади неправильной формы разделите форма с кривыми в две или более правильные формы.

    В этом методе разделите неправильную форму на несколько квадратов, треугольников или других четырехугольников. В зависимости от формы или кривых, часть фигуры может быть кругом, полукругом или квадрантом.

    Найдите участки заданной неправильной формы с 8 сторонами, включая одну кривую.

    Sol: Мы определим неизвестные количества по заданным размерам сторон. Для начала нам нужно разделить фигуру на два прямоугольника и полукруг.

    Площадь фигуры ABCDEF составляет:

    Общая площадь (ABCDEF) = Площадь (ABCG) + Площадь (GDEF) + Площадь (aob)

    Общая площадь = (AB × AG) + (GD × DE) + (1⁄2 × π × ob 2 )

    Общая площадь = (3 × 4) + (10 × 4) + (1⁄2 × 3.14 × 12)

    Общая площадь = 12 + 40 + 1,57

    Следовательно, общая площадь = 53,57 см 2

    Как найти площадь неправильной формы с помощью миллиметровой бумаги?

    Какова площадь неровной поверхности?

    Найдите площадь данного листа.

    Решение: Чтобы найти площадь неровной поверхности в листе вышеупомянутого случая, мы должны поместить лист на миллиметровую бумагу и нарисовать его границу.

    Форма листа неправильная.Итак, мы предположим, что более половины квадрата, покрытого листом, будет считаться за 1, а меньшее — за 0.

    Теперь подсчитайте количество полностью закрытых фигур. Полностью покрыто 64 квадрата.

    Также посчитайте частично покрытые более чем половиной квадраты, и каждый будет считать qs 1 квадрат. Есть 17 кв. Больше половины кв.

    Также посчитайте частично менее чем половину покрытого квадрата, и каждый будет засчитан как 0.Есть 16 квадратов меньше половины квадрата.

    Теперь сложите все квадраты, чтобы найти площадь листа = 64 + 17 x 1 + 16 x 0 = 64 + 17 = 81 квадратную единицу.

    Следовательно, площадь створки составит 81 кв.

    Формула площади неправильных форм

    Чтобы найти площадь неправильной формы, во-первых, нам нужно разделить неправильную форму на правильные формы, которые вы можете распознать, такие как треугольники, прямоугольники, круги, квадраты и т. Д.

    Затем найдите площадь этих отдельных форм и сложите их, чтобы получить область неправильных форм.

    Решенный пример

    В. Найти площадь данной формы?

    Sol: Фигура выше имеет три правильные формы. Начинайте деление сверху, у него есть треугольник, прямоугольник и трапеция.

    Мы найдем площадь для каждой из этих трех фигур и сложим результаты, чтобы получить окончательную площадь фигуры.

    Треугольник

    Площадь треугольника = (основание × высота) / 2

    = (3 × 4) / 2

    = 12/2

    = 6

    Прямоугольник

    Площадь прямоугольника = длина × ширина

    = 3 × 10

    = 30

    Трапеция

    Площадь трапеции = ((b 1 + b 2 ) × h) / 2

    = ((3 + 5) × 2) / 2

    = (8) × 2/2

    = 16/2

    = 8

    Следовательно, площадь данной формы = 6 + 30 + 8 = 44.

    Как рассчитать площадь неправильной формы

    Обновлено 17 ноября 2020 г.

    Лиза Мэлони

    Когда вы впервые начинаете вычислять площадь, вы получаете простые формы с четко определенными формулами для определения площади: круги, треугольники, квадраты и прямоугольники, например. Но что происходит, когда вы сталкиваетесь с фигурой, которая не вписывается в эти категории? Пока вы не войдете в дивный новый мир математических интегралов, лучший способ найти область неправильных форм — это разделить их на формы, с которыми вы уже знакомы.

    TL; DR (слишком долго; не читал)

    Самый простой способ вычислить площадь неправильной формы — разделить ее на знакомые формы, вычислить площадь знакомых форм, а затем суммировать эти вычисления площади, чтобы получить площадь неправильной формы, которую они составляют.

      Соберите формулы площади для форм, с которыми вы уже знакомы. Наиболее распространенные формы и их формулы включают:

      \ text {Площадь квадрата или прямоугольника} = l × w

      , где l — длина, а w — ширина.2

      , где r — радиус окружности.

      Используйте свое воображение, чтобы разделить имеющуюся неправильную форму на более знакомые формы. Иногда рисование формы с последующим добавлением линий для подразделений помогает визуализировать ее и отслеживать соответствующие измерения для каждого измерения. Например, представьте, что вам нужно найти область пятиугольной формы, которая не является шестиугольником, но имеет три перпендикулярные стороны, противоположные «точке». Немного подумав, вы можете разделить это на прямоугольник, который стыкуется с треугольником, причем треугольник образует «точку» формы.

      Вернитесь к формулам площади, чтобы узнать размеры, которые вам понадобятся для расчета площади каждой разделенной фигуры. В этом случае вам понадобятся базовая и вертикальная высота треугольника, а также длина и ширина (или две смежные стороны) прямоугольника. Если вы решаете математическую задачу в школе, вы, вероятно, получите по крайней мере некоторые из этих измерений и, возможно, потребуется использовать некоторую базовую алгебру или геометрию, чтобы найти любые недостающие измерения. Если вы работаете в реальном мире, возможно, вы сможете заполнить некоторые измерения путем физического измерения.2

    Что такое площадь неправильной формы?

    Участок неправильной формы

    Неправильные формы — это многоугольники с пятью или более сторонами различной длины. Эти формы или фигуры можно разложить на треугольники, квадраты и четырехугольники для оценки площади.

    Вот некоторые примеры неправильной формы:

    Предметы повседневного быта неправильной формы

    Расчет площади неправильной формы:

    Подходы к оценке площади неправильной формы:

    Оценка площади с использованием единичных квадратов

    Примените эту технику для фигур с кривыми, кроме идеального круга или полукругов и неправильных четырехугольников.В этом методе разделите фигуру на единичные квадраты. Общее количество единичных квадратов, попадающих в форму, определяет общую площадь.

    Рисунок: Некоторые примеры неправильной формы

    Посчитайте квадрат как «1», если заштрихованная область покрывает более половины при вычислении площади для более точной оценки.

    Рис.: Для фигур неправильной формы считайте квадраты с оранжевой и желтой кодировкой как 1.

    На следующем рисунке вычислите площадь, подсчитав единичные квадраты, что равно 6.Если обозначить каждую единицу квадрата в сантиметрах, то площадь будет 6 см2.

    Рис.: Расчет площади неправильной формы с изогнутыми краями

    • Разделение неправильной формы на две или более правильных формы

    Используйте этот метод для неправильных форм, которые представляют собой комбинацию треугольников и многоугольников. Используйте стандартные формулы, чтобы вычислить площадь таких фигур и сложить их вместе, чтобы получить общую площадь.

    Например, у неправильной формы мы разделяем несколько ребер на треугольник и три многоугольника.

    Общая площадь фигуры определяется как:

    ⇒ Площадь = Площадь (ABIM) + Площадь (BCGH) + Площадь (CDEF) + Площадь (JKL)

    ⇒ Площадь = (AB × BI) + (BC × CG) + (CD × DE) + ( 1 2 × LJ × KO)

    ⇒ Площадь = (10 × 5) + (3 × 3) + (2 × 2) + ( 1 2 × 4 × 4)

    ⇒ Площадь = 50 + 9 + 4 + 8

    ⇒ Площадь = 71 см2

    • Разделение неправильной формы кривыми на две или более правильных формы

    В этом методе разложите неправильную форму на несколько квадратов, треугольников или других четырехугольников.В зависимости от формы и кривых, часть фигуры может быть кругом, полукругом или квадрантом.

    На следующем рисунке изображена неправильная форма с 8 сторонами, включая одну кривую. Определите неизвестные величины по заданным размерам сторон. Разложите фигуру на два прямоугольника и полукруг.

    Площадь фигуры ABCDEF составляет:

    Площадь (ABCDEF) = Площадь (ABCG) + Площадь (GDEF) + Площадь (aob)

    Площадь = (AB × AG) + (GD × DE) + ( 1 2 × π × ob2)

    Площадь = (3 × 4) + (10 × 4) + ( 1 2 × 3.14 × 12)

    Площадь = 12 + 40 + 1,57

    Площадь = 53,57 см2

    Приложение

    Оценка площади неправильных фигур — важный метод для рисования карт, построения архитектуры и разметки сельскохозяйственных полей. Применяем концепцию раскроя тканей по заданному дизайну. В старших классах эта техника закладывает основу для таких продвинутых тем, как вычисление объема, рисование конических сечений и фигур эллиптической формы.

    Интересный факт

    Сопутствующий математический словарь
    • Квадрат
    • Прямоугольник
    • Треугольник
    • Круг
    • Площадь
    • Неправильные и правильные формы

    5.РАСЧЕТ ПОВЕРХНОСТИ ПОЛЕЙ НЕРЕГУЛЯРНОЙ ФОРМЫ

    5. РАСЧЕТ ПОВЕРХНОСТИ ПОЛЕЙ НЕРЕГУЛЯРНОЙ ФОРМЫ



    5.1 Пример 1
    5.2 Пример 2

    Распространенной проблемой для геодезистов является расчет площади поверхности поле фермера. Поля часто нерегулярны, что позволяет производить прямые вычисления. из их районов сложно. В таком случае поля делятся на ряд обычных области (треугольники, прямоугольники и т. д.), из которых можно рассчитать поверхности с простыми формулами. Все площади рассчитываются отдельно, и их сумма area дает общую площадь поля.

    На рисунке 29 показано поле неправильной формы, площадь поверхности которого необходимо определить.

    Рис.29 Поле неправильной формы

    Следующая процедура:

    Шаг 1

    Сделайте грубый набросок поля (см. Рис.29а) с указанием углов поля (A, B, C, D и E) и границ поля (прямые линии). Вдобавок какой-то крупный ориентир! обозначены (дороги, канавы, дома, деревья и т. д.), которые могут помочь определить местонахождение поля.

    Рис. 29a Черновой набросок поля

    Шаг 2

    Разделите поле, как показано на эскизе, на области правильной формы. В этом примере поле можно разделить на 3 треугольника ABC (основание AC и высота BB,), AEC (основание AC и высота EE 1 ) и CDE (основание EC и высота DD 1 ) (см. Рис.29b).

    Рис. 29б Разделение поля на области правильной формы

    Шаг 3

    Отметьте на поле углы A, B, C, D и E колышками.

    Шаг 4

    Разместите полюса измерения дальности на линиях AC (основание треугольников ABC и AEC) и EC (основание треугольника EDC) (см. Рис. 29c) и измерьте расстояния AC и EC.

    Рис. 29c Отметьте углы колышками и разложите мачты

    Шаг 5

    Проведите линию BB (высота треугольника ABC) перпендикулярно базовой линии AC (см. Рис.29d) одним из методов, описанных в главе 4. Измерьте расстояние BB,

    Рис. 29d Проведите линию BB перпендикулярно к AC

    Шаг 6

    Таким же образом устанавливаются и измеряются высота EE треугольника AEC и высота DD треугольника CDE (см. Рис. 29e).

    Рис. 29e Проведите линию DD 1 перпендикулярно EC и линию EE1 перпендикулярно AC

    Шаг 7

    Основание и высота трех треугольников были измерены.Окончательный расчет можно произвести следующим образом:

    Измерено

    Треугольник ABC: основание = AC = 130 м
    высота = BB 1 = 55 м
    Треугольник ACE: основание = AC = 130 м
    высота = EE 1 = 37 м
    Треугольник CDE: основание = EC = 56 м
    высота = DD 1 = 55 м

    Ответ

    Площадь = 0,5 x основание x высота
    = 0,5 x 130 м x 55 м = 3 575 м 2

    Площадь = 0.5 x 130 м x 37 м = 2 405 м
    Площадь = 0,5 м x 56 м x 55 м = 1 540 м²

    Поле ABCDE:

    Площадь треугольника ABC = 3 575 м 2
    Площадь треугольника ACE = 2 405 м 2
    Площадь треугольника CDE = 1540 м 2

    Общая площадь = 3 575 м 2 + 2 405 м 2 + 1540 м 2
    = 7 520 м- = 0,752 га

    Площадь поля, показанного на рис.30, должна определяться в то время, когда поле покрыто высокой культурой (например.грамм. кукуруза или сахарный тростник).

    Рис.30 Поле, покрытое высоким урожаем

    Поле можно разделить на два треугольника ABD и BCD (см. Рис. 31a). К сожалению, из-за высокого урожая невозможно установить и измерить базовый BD и две высоты AA 1 и CC 1 .

    Рис. 31a Разделение поля на два треугольника

    В этом случае площадь треугольника ABD может быть рассчитана с использованием AD в качестве основания и BB 1 в качестве соответствующей высоты.BB 1 можно разметить и измерить за пределами посевной площади. Таким же образом можно рассчитать треугольник BCD, используя основание BC и соответствующую высоту DD 1 (см. Рис. 31b).

    Рис. 31b Определение площадей двух треугольников

    Порядок действий на поле:

    Шаг 1

    Отметьте 4 угла (A, B, C и D) с помощью опорных стоек.

    Шаг 2

    Линия AD проходит с дальнобойными мачтами и проходит за A.Линия BC также проходит и продолжается за C (см. Рис. 32a). Измерьте расстояния AD (основание треугольника ADB) и BC (основание треугольника BCD).

    Рис. 32a Измерение оснований двух треугольников

    Шаг 3

    Проведите линию BB 1 (высота треугольника ABD) перпендикулярно расширенной базовой линии AD, используя один из методов, описанных в главе 4. Таким же образом устанавливается линия DD 1 (высота треугольника BCD). перпендикулярно удлиненной базовой линии BC (см. рис.32b) Измерьте расстояние BB 1 и DD 1 .

    Рис. 32b Измерение высоты двух треугольников

    Шаг 4

    Основание и высота обоих треугольников были измерены. Окончательные расчеты можно произвести следующим образом:

    Измерено

    Треугольник ABD: основание = AD = 90 м
    высота = BB 1 -37 м

    Треугольник BCD: основание = BC = 70 м
    высота = DD 1 -50 м

    Ответ

    Площадь = 0.5 x основание x высота
    = 0,5 x 90 м x 37 м = 1665 м 2

    Площадь = 0,5 x 70 м x 50 м = 1750 м 2

    Поле ABDC:

    Треугольник площади ABD = 1 665 м²
    Треугольник площади BCD = 1750 м 2

    Общая площадь = 1665 м 2 + 1750 м 2 = 3415 м 2
    = 0,3415 га = ок. 0,34 га


    Как найти площадь неправильных многоугольников — стенограмма видео и урока

    Правильные и неправильные многоугольники

    Площадь неправильного многоугольника

    Чтобы найти площадь неправильного многоугольника, вы должны сначала разделить фигуру на правильные многоугольники или плоские формы.Затем вы используете формулы площади правильного многоугольника, чтобы найти площадь каждого из этих многоугольников. Последний шаг — сложить все эти области вместе, чтобы получить общую площадь неправильного многоугольника.

    Давайте рассмотрим несколько формул для определения площади многоугольников. Помните, что:

    b = основание

    h = высота

    a = длина стороны

    w = ширина

    Площадь треугольника = 1/2 x b x h

    Площадь квадрата = a 2

    Площадь прямоугольника = w x h

    Площадь параллелограмма = b x h

    Пример 1

    Начнем с следующая форма:

    Неправильный многоугольник

    Если мы разделим или разделим эту форму, она будет выглядеть так:

    Разрезанный неправильный многоугольник

    Мы должны использовать формулу для прямоугольников, поскольку неправильный многоугольник был разделен на три разных прямоугольника.Первый прямоугольник имеет ширину 4 см и высоту 6 см. По формуле нам нужно 4 см умножить на 6 см. Наш продукт будет 24 см2. Итак, площадь первого прямоугольника составляет 24 см2. Вот формула:

    Площадь = w x h = 4 см x 6 см = 24 см2

    Мы возводим произведение в квадрат, потому что мы умножаем два разных измерения: высоту и ширину плоской формы. Теперь переходим ко второму прямоугольнику шириной 5 см и высотой 4 см.Используя ту же формулу:

    Площадь = ширина x высота = 5 см x 4 см = 20 см2

    Третий прямоугольник имеет ширину 4 см и высоту 9 см. Формула выглядит следующим образом:

    Площадь = w x h = 4 см x 9 см = 36 см2

    Последний шаг — сложить произведения всех формул для всех прямоугольников. Это даст нам площадь всего неправильного многоугольника. Поскольку он был разделен на три разных прямоугольника, мы складываем все три продукта.

    24 см2 (прямоугольник 1) + 20 см2 (прямоугольник 2) + 36 см2 (прямоугольник 3) = 80 см2

    Пример 2

    Мы можем использовать эту технику, чтобы найти площадь любого неправильного многоугольника. Представим, что у нас есть неправильный многоугольник, разделенный на треугольник, квадрат и прямоугольник. Треугольник имеет основание 6 см и высоту 12 см. Квадрат имеет длину одной стороны 6 см. Прямоугольник имеет ширину 18 см и высоту 3 см.

    Формула для определения площади треугольника: площадь = 1/2 x основание x высота.Подставьте измерения:

    Площадь = 1/2 x 6 см x 12 см = 3 см x 12 см = 36 см2

    При определении площади треугольника необходимо умножить его вдвое, поскольку треугольник равен половине параллелограмма. . Вы также можете разделить на два. Товар будет таким же.

    Площадь этого треугольника 36 см2.

    Теперь по площади. Поскольку у квадрата все стороны одинаковой длины, формула просто требует, чтобы мы возводили в квадрат длину одной стороны, что равносильно умножению ширины и высоты.

    Площадь = длина одной стороны = 6 см = (6 см) 2 = 6 см x 6 см = 36 см2.

    И, наконец, прямоугольник. Формула:

    Площадь = 18 см x 3 см = 54 см2

    Теперь, когда у нас есть площадь для каждой формы, мы должны сложить их вместе и получить формулу для всего многоугольника.

    36 см2 (треугольник) + 36 см2 (квадрат) + 54 см2 (прямоугольник) = 126 см2

    Итак, этот неправильный многоугольник имеет площадь 126 см2.

    Резюме урока

    Неправильные многоугольники — это многоугольники, которые не имеют равных сторон или равных углов.При нахождении площади неправильного многоугольника разделите многоугольник на более мелкие правильные многоугольники. Затем вы можете вставить эти измерения в соответствующие формулы площади и умножить, чтобы найти площадь одной части формы. Когда у вас есть площадь каждой отдельной формы, сложите их вместе, чтобы получить площадь всего многоугольника.

    Оценка площади неправильных форм — видео и стенограмма урока

    План этажа спальни

    Что такое площадь?

    Площадь — это объем пространства внутри фигуры.Представьте, что у вас есть прямоугольник и множество квадратов в один дюйм, вырезанных из плотной бумаги. Количество квадратов в один дюйм, необходимое для того, чтобы полностью покрыть прямоугольник без зазоров или перекрытий, и есть площадь прямоугольника. Если бы для покрытия прямоугольника потребовалось 42 из этих квадратов в один дюйм, то этот прямоугольник имел бы площадь 42 квадратных дюйма. Площадь всегда измеряется в квадратных единицах , таких как квадратные дюймы, квадратные сантиметры, квадратные футы и т. Д. Она также записывается как 42 кв. Дюйма или 42 кв. Дюйма.

    Площадь основных фигур

    Чтобы найти площадь фигуры, вы можете покрыть ее множеством квадратов, а затем посчитать, но это занимает много времени. Есть более быстрый способ. Формулы позволяют быстро находить области общих форм. Все, что вам нужно сделать, это провести пару измерений и произвести некоторые вычисления. Вам неоднократно давали измерения, что облегчало задачу. Одна из самых простых формул — это площадь прямоугольника. Площадь прямоугольника равна длине, умноженной на ширину ( A = lw ).Если вы знаете длину и ширину прямоугольника, вы умножаете их вместе, чтобы получить площадь.

    Другие распространенные формы также имеют формулы. Формула для определения площади треугольника: площадь = основание, умноженное на высоту, деленное на 2 ( A = bh /2), а формула для определения площади круга: площадь = 3,14, умноженная на квадрат радиуса (3,14 r 2).

    Области основных форм

    Область неправильных форм

    Многие фигуры не являются базовыми фигурами, такими как прямоугольники, треугольники и круги, для которых у нас есть формулы.Бывают случаи, когда вам нужно найти область формы, которая не является правильной формой. Один из способов найти площадь неправильной формы — разделить форму на более мелкие формы, для которых у вас есть формула. Затем вы находите области всех меньших фигур и складываете все свои области вместе.

    Разделение неправильных форм на общие

    Давайте посмотрим на этот пример пятиугольника.У нас нет формулы для определения площади пятиугольника; однако, если мы разделим пятиугольник на прямоугольник и треугольник, у нас будут формулы для обеих этих форм. В данном случае размер прямоугольника составляет 10 x 12, следовательно, его площадь составляет 120 кв. Футов. Треугольник имеет основание 10 и высоту 3, и, используя нашу формулу, (10 x 3) / 2 составляет 15 футов2. Сложив эти две области вместе, мы обнаружим, что площадь нашего пятиугольника составляет 135 кв. Футов.

    Иногда вы обнаруживаете область неправильной формы, которая кажется формой внутри другой формы.В этом случае вы отбираете какую-то площадь. Вы должны вычесть площадь внутренней формы из площади внешней формы.

    В этом примере мы хотим найти площадь нашего круга, а затем вычесть площадь прямоугольника, который находится внутри круга. Площадь круга определяется умножением на 3,14 квадрата радиуса (8), так что площадь круга составляет 200,96 кв. Футов; а площадь прямоугольника вычисляется путем умножения 12 на 2, так что площадь прямоугольника составляет 24 кв. фута.Таким образом, площадь неправильной формы определяется путем вычисления 200,96–24, что равно 176,96 кв. Фута.

    Иногда у вас также могут быть неправильные формы, которые не создаются формами, для которых у вас есть формулы. Когда это произойдет, вам нужно будет использовать приблизительные значения. Покройте неправильную форму основными формами как можно лучше. У вас будет слишком много покрытия в одной области и недостаточно в другой. Это будет средним, чтобы дать приличное приближение для области.

    Приближение площади

    В этом примере мы не можем разделить фигуру на наши простые в использовании фигуры; вместо этого мы накладываем на неправильную форму известные формы, а затем вычисляем площади и складываем их вместе.Это даст нам приблизительное значение. В этом случае мы делим фигуру на 3 прямоугольника разного размера, а затем складываем их области вместе.

    Прямоугольник 1 равен 2 x 10, что равно 20 кв.

    Прямоугольник 2 имеет размер 4,5 x 6, что равно 27 кв.

    Прямоугольник 3 имеет размер 1,5 x 8, что равно 12 кв.

    Сложив все три площади вместе, мы получим 20 + 26 + 12, что в сумме составит 59 кв. Футов.

    Резюме урока

    Нам часто приходится определять площадь форм.Основные формы, такие как прямоугольник, треугольник и круг, имеют формулы, которые действуют как ярлыки для поиска области. Если у вас неправильная форма, вам нужно будет использовать различные приемы. Вы можете сложить области двух или более фигур вместе, чтобы найти область. Возможно, вам придется вычесть площадь одной основной формы из площади более крупной. В других случаях неправильная форма не может быть получена из основных форм с формулами. Затем вам нужно будет сделать приближение, используя основные формы.

    Нахождение неправильных фигур

    Результаты обучения

    • Объедините области правильных форм, чтобы найти области неправильных форм.

    Итак, мы нашли область для прямоугольников, треугольников, трапеций и кругов. Неправильная фигура — это фигура, не имеющая стандартной геометрической формы. Его площадь не может быть рассчитана ни по одной из стандартных формул площади. Но некоторые неправильные фигуры состоят из двух или более стандартных геометрических фигур. Чтобы найти площадь одной из этих неправильных фигур, мы можем разбить ее на фигуры, формулы которых нам известны, а затем сложить площади фигур.

    пример

    Найдите площадь заштрихованной области.

    Решение
    Данный рисунок неправильный, но мы можем разбить его на два прямоугольника. Площадь заштрихованной области будет суммой площадей обоих прямоугольников.


    Синий прямоугольник имеет ширину [латекс] 12 [/ латекс] и длину [латекс] 4 [/ латекс]. Красный прямоугольник имеет ширину [латекс] 2 [/ латекс], но его длина не указана. Правая часть рисунка — это длина красного прямоугольника плюс длина синего прямоугольника. Поскольку правая сторона синего прямоугольника имеет длину [латекс] 4 [/ латекс] единиц, длина красного прямоугольника должна быть [латекс] 6 [/ латекс] единиц.


    Площадь фигуры [латекс] 60 [/ латекс] квадратных единиц.
    Есть ли другой способ разбить эту фигуру на два прямоугольника? Попробуйте и убедитесь, что у вас такая же площадь.

    В следующем видео показан еще один пример того, как найти площадь многоугольника L-образной формы, используя размеры двух прямоугольников.

    пример

    Найдите площадь заштрихованной области.

    Показать решение

    Решение
    Мы можем разбить эту неправильную фигуру на треугольник и прямоугольник.Площадь фигуры будет суммой площадей треугольника и прямоугольника.
    Прямоугольник имеет длину [латекс] 8 [/ латекс] единиц и ширину [латекс] 4 [/ латекс] единиц.
    Нам нужно найти основание и высоту треугольника.
    Поскольку обе стороны прямоугольника [латекс] 4 [/ латекс], вертикальная сторона треугольника — [латекс] 3 [/ латекс], то есть [латекс] 7 — 4 [/ латекс].
    Длина прямоугольника составляет [латекс] 8 [/ латекс], поэтому основание треугольника будет [латекс] 3 [/ латекс], то есть [латекс] 8 — 4 [/ латекс].


    Теперь мы можем добавить области, чтобы найти площадь неправильной фигуры.


    Площадь фигуры [латекс] 36,5 [/ латекс] квадратных единиц.

    пример

    Трасса средней школы имеет форму прямоугольника с полукругом (полукругом) на каждом конце. Прямоугольник имеет длину [латекс] 105 [/ латекс] метров и ширину [латекс] 68 [/ латекс] метров. Найдите область, ограниченную дорожкой. Округлите ответ до ближайшей сотой.

    Добавить комментарий

    Ваш адрес email не будет опубликован. Обязательные поля помечены *